uworld questions I got wrong

Ace your homework & exams now with Quizwiz!

A research lab finds that 95% of a test are negative for prostate cancer in patients who do not have the disease. If the new assay is used on 8 blood samples taken from patients without prostate cancer, what is the probability of all 8 tests coming back negative? A. 0.95^8 B. 1-0.05^8

A. 0.95^8 If events are independent, the probability of all events turning out the same is the product of all separate probabilities for each event. The probability of at least 1 even turnign out differently is given as 1 - P(all events being the same).

Researchers are studying coronary circulation. They measure blood flow and pressure (blue circle) in one of the epicardial coronary arteries of an experimental animal with CV physiology similar to that of humans. They then clamp the artery to stimulate 60% atherosclerotic stenosis and remeasure the flow and pressure distal to the lesion. Which of the following changes are most likely to be observed once steady state is achieved? https://i.imgur.com/TfwloH2.png A. D B. C

A. D Perfusion pressure decreases distal to a significant coronary artery stenosis, resulting in an initial decrease in blood flow. Coronary autoregulation stimulates arteriolar vasodilation, which reduces downstream vascular resistance and helps return blood flow to near-normal levels to prevent myocardial ischemia. https://i.imgur.com/371fHil.png

A mass is discovered in a 70-year-old woman's head. When she comes in for a follow-up, her son comes along and asks you not to reveal the results to his mother. What is the best response to the patient's son? A. I appreciate you sharing this with me; I'd like to hear more about why you prefer that your mother does not know about the results B. I understand you're concerned about your mother, but its up to her to decide whether she would like to know about the results

A. I appreciate you sharing this with me; I'd like to hear more about why you prefer that your mother does not know about the results Autonomy, supprting a patient's right to know his or her diagnosis, is a fundamental ethical principle. However, limiting disclosure of a diagnosis may be appropriate under certain circumstances (eg, patient preference, imminent safety concerns). An understanding, open approach is necessary to uncover these circumstances. My choice expresses empathy, but does not give the son a chance to explain why he opposes disclosing the diagnosis. Premature dismissal risks the loss of valuable information (like safety concerns)

A 39-year old, nulliparous woman has pelvic pressure and constipation that has worsened over the past year. She has frequent uncomfortable sensations of incomplete evacuation following defecation. Menses occur every 28 days without heavy bleeding or severe pain. She has no pain with intercourse and uses condoms regularly. Exam shows irregularly enlarged uterus and normal rectal tone. What is the most likely etiology of the patient's constipation? A. Posterior subserosal uterine leiomyoma B. adenomyosis

A. Posterior subserosal uterine leiomyoma Subserosal uterine leiomyoma (fibroids) can cause irregular uterine enlargement and bulk-related symptoms (pelvic pressure). Posterior leiomyomas can cause constipation due to pressure on the colon. Adenomyosis typically presents with dysmenorrhea, heavy menstrual bleeding, and a diffusely tender, uniformly globular uterus rather than an irregularly enlarged uterus. It is not associated with constipation.

A 42-year-old man has an episode of syncope. He was standing in the kitchen when he felt palpitations for 3 seconds. The next thing he remembers is waking up on the floor. He has had intermittent palpitations for the last few weeks, sometimes with associated lightheadedness. He has no significant medical history other than chronic lower back pain that began following a fall from a ladder 3 years ago. The patient fractured several lumbar vertebrae in that fall and required a surgical lumbar fusion. His back pain was uncontrolled with conservative measures, and he has been treated with escalating doses of methadone for the past year. He has a remote history of IV heroin abuse but has not used it for 20 years. BP is 120/70 when supine and 125/75 when standing. Which EKG finding is most likely? A. QT interval prolongation B. AV block

A. QT interval prolongation Methadone slows the delayed rectifier potassium current responsible for ventricular repolarization, and high doses are associated with QT interval prolongation. QT interval prolongation predisposes to the development of torsade de pointes, a serious ventricular arrhythmia that can cause syncope and sudden cardiac death. Advanced AV block (Mobitz type II or complete AV block) can present with syncope. Common causes include infection (Lyme disease), ischemia, medications (beta blockers, nondihydropyridine CCBs). Methadone is not associated with AV block.

A 36-year-old woman has a rash for the past 3 days that started on the torso and spread to the extremities. She has not had a similar rash previously. The patient feels mildly feverish with no other symptoms. Medical history is notable for multiple sclerosis and associated trigeminal neuralgia. Medications include a recombinant anti-B-cell antibody that was started a year ago and carbamazepine for the past 2 weeks. Temp is 100.4. Exam shows a rash on the best, abdomen, back, and proximal extremities as shown here: https://i.imgur.com/OKEmvJt.png. Conjunctivae are clear bilaterally, and oropharyngeal exam is unremarkable. Rash is most likely caused by what? A. adverse drug reaction B. scarlet fever

A. adverse drug reaction Morbilliform drug eruption is a type IV hypersensitivity reaction caused by drugs (anticonvulsants, antibiotics) or their metabolites that typically occur 5-21 days following drug initiation. Lesions consist of erythematous macules and papules, distributed symmetrically on the trunk and extremities, sometimes accompanied by low-grade fever. Mucosal involvement is absent. Scarlet fever is associated with streptococcal pharyngitis and presents with diffuse, fine, erythematous papules with a sandpaper-like texture. The patient does not have sore throat and lacks tonsilitis on exam.

A 16-year-old boy is brought to the ED with severe right groin pain that has worsened over the past 12 hours. During the last month, he had several episodes of mild scrotal pain while walking between classes at school. He has no history of traumatic injury and is sexually active. Medical history is unremarkable except for an inguinal hernia repaired a year ago. On exam, the right hemiscrotum is swollen and tender. The bisected gross specimen from an orchiectomy is shown here: https://i.imgur.com/jvuEvX5.png. What is the most likely mechanism of this acute testicular pain? A. anatomic defect causing increased mobility of the testis B. postsurgical clotting of the pampiniform plexus

A. anatomic defect causing increased mobility of the testis Testicular torsion is characterized by spermatic cord twisting due to an anatomic defect that allows increased testicular mobility. The presentation includes severe scrotal pain and swelling due to venous compression and hemorrhagic infarction of the testis. Postoperative clotting of the pampiniform plexus (thrombotic varicocele) is a rare surgical complication that causes acute scrotal pain and swelling. However, this patient's inguinal hernia repair was a year ago, making postsurgical complication unlikely.

A 34-year-old woman comes to the ED due to 6 days of fatigue and intermittent fever. Hx is significant for depression. She lives in New York City and recently traveled to central Africa. Blood microscopy shows ring-form trophozoites within RBCs. What is the best treatment for this patient? A. artemisinin B. pyrimethamine

A. artemisinin Small rings within RBCs suggests Plasmodium infection (malaria). Erythrocytic forms of this parasite are treated with antimalarials such as chloroquine (in nonresistant areas), atovaquone-proguanil, or artemisinins. Primaquine is required to kill Plasmodium vivax and Plasmodium ovale liver hypnozoites. Most species of malaria are resistant to pyrimethamine now, but it can be used to treat toxoplasmosis.

A 47-year-old man has persistent fever, night sweats, and fatigue. Evalulation yields diagnosis of chronic myeloid leukemia. While undergoing chemo, the patient experiences headaches, scant nasal discharge, and left eye symptoms. Physical exam shows tenderness over the paranasal sinuses and left-sided orbital swelling and cellulitis. Mild proptosis and ptosis of the left eye are present. Biopsy of sinus mucosa: https://i.imgur.com/TibUjJ8.png. What is the most likely cause of this patient's condition? A. aspergillus fumigatus B. cryptococcus neoformans

A. aspergillus fumigatus Immunosuppressed patients are at risk for Aspergillus fumigatus infection. This fungus produces thin, septate hyphae with acute, V-shaped branching, causing invasive aspergillosis, aspergillomas, and allergic bronchopulmonary aspergillosis. C. neoformans is an oval, budding yeast with a thick capsule causing meningitis and lung infections in AIDS. India ink of CSF reveals wide, clear zone around unstained capsule.

A 14-year-old athlete is evaluated due to left hip and thigh pain. 3 days ago, the patient felt a sudden pain in his left hip and thigh. He can ambulate, but it is very painful. He has been unable to run since his injury. On exam, active hip extension and knee flexion are limited by pain. Gait is antalgic. X-rays of the hip and pelvis reveal an avulsion fracture of the ischium. Injury to which muscle is most likely responsible for this patient's pain? A. biceps femoralis B. iliopsoas

A. biceps femoralis The hamstrings are composed of the biceps femoralis (long and short heads), semitendinosus, and semimembranosus. Hamstring injury can result in painful, limited hip extension and knee flexion, and ischial avulsion fractures can occur at origin sites. The iliopsoas, comprising the iliacus, psoas major, and psoas minor, originates at the T12-L4 vertebrae and pelvic brim and inserts on the lesser trochanter of the femur. Iliopsoas injury or irritation (abscess) causes pain with active hip flexion (vs extension); knee movement is not affected because the iliopsoas does not cross the knee joint.

A 26-year-old woman has recent weight gain. She has eaten more than usual over the last 5 months, gaining 7 lbs, and she feels guilty and depressed about it. She reports consuming a large pizza and two large bags of chips multiple times a week. Afterwards, she feels ashamed about being unable to control her intake and fasts to make up for it. She is very distressed about being unable to lose weight despite excercising 2-3 hours a day. BMI is 23.7, but she insists that she is overweight. What is the most likely diagnosis? A. bulimia nervosa B. anorexia nervosa

A. bulimia nervosa Bulimia nervosa (BN) is characterized by recurrent episodes of binge eating followed by compensatory behaviors to prevent weight gain (fasting, exercising, vomiting, using laxatives). In contrast to patients with anorexia nervosa, BN patients are normal weight or overweight. Although both BN and anorexia nervosa may be characterized by preoccupation with weight, binge eating, and compensatory behavior (there is a binge-eating/purging subtype of anorexia). Patients with anorexia nervosa have significantly lower body weight (BMI < 18.5).

A 15-year-old boy fell while skateboarding 3 weeks ago and sustained a large abrasian to his right flank. Since then, he has been showering daily and applying topical antibiotic ointment to the affected area. Exam of his right flank today shows a clean, healed wound covered by pearly pink epithelium. The cells responsible for repopulation of this patient's epithelium are most likely to demonstrate which of the following features? A. cellular proliferation controlled by contact inhibition B. melanosome production and transfer to adjacent cells

A. cellular proliferation controlled by contact inhibition Keratinocytes are responsible for wound reepithelization. They migrate into the wound from its edges and are repopulated by replication within the stratum basale. Keratinocytes contrinue to migrarte and proliferate until they contact other similar cells, a regulatory mechanism known as contact inhibition. Melanocytes are melanin-producing cells within the stratum basale that are derived from NCCs. Failure of melanocytes to migrate into a healing scar can lead to permanent scar hypopigmentation.

A 64-year-old man has acute-onset RUQ pain, N/V. He had extensive small bowel resection due to bowel ischemia a year ago and has been receiving parenteral nutrition since then. His other medical problems include Afib and hypertension. Exam shows RUQ tenderness on deep palpation. Labs show moderate leukocytosis with normal liver enzymes. Abdominal US reveals gallstones and edema of the gallbladder wall. Review of prior records indicates that he had no gallstones noted on abdominal imaging performed prior to the bowel resection. Which of the following is most likely responsible for the development of gallstones in this patient? A. decreased CCK release due to lack of enteral stimulation B. high cholesterol content of the nutritional fluids

A. decreased CCK release due to lack of enteral stimulation Absence of normal enteral stimulation in patients receiving total parenteral nutrition leads to decreased CCK release, biliary stasis, and increased risk of gallstones. Resection of the ileum can also increase the risk of gallstones due to disruption of normal enterohepatic circulation of bile acids. Serum cholesterol would not be significantly altered.

An 80-year-old woman is evaluated for blisters that have developed on both feet over the last week. The patient lives in a nursing home due to dementia but has no other chronic medical conditions and takes no medication. There are no oral lesions. Exam findings:https://i.imgur.com/h7ZBMa2.png. Which of the following is most likely involved in the pathogenesis of this condition? A. deposition of IgG and complement at the dermal-epidermal junction B. disrupted assembly of keratin into monofilaments

A. deposition of IgG and complement at the dermal-epidermal junction Bullous pemphigoid is an autoimmune blistering disease that typically occurs in patients >60. It presents with tense bullae with rare mucosal involvement. Autoantibodies against hemidesmosomes create subepidermal cleavage. Immunofluorescence shows linear deposition of IgG and/or C3 along the basement membrane. Disrupted assembly of keratin into monofilaments describes epidermolysis bullosa simplex, an autosomal dominant condition presenting with blisters in areas of high friction (hands, feet, knees). Unlike BP, the condition typically presents in infancy through early adulthood.

A 68-year-old male has difficulty urinating. He had prostate cancer treated with radiation therapy a year ago. US reveals bilateral dilation of the ureters and renal calyces. What is the most likely cause of this patient's condition? A. fibrosis B. radiation mucositis

A. fibrosis Fibrosis and strictures are late effects of radiation therapy. Radiotherapy for prostate cancer may lead to urethral fibrosis and result in obstructive uropathy. Radiation mucositis occur after radiotherapy for GI tumors and head and neck malignancies. These early effects resolve soon after discontinuation of treatment.

A 72-year-old woman has persistent headaches, fatigue, and muscle pain. She has a history of hypertension and osteoarthritis. She does not use tobacco, alcohol, or recreational drugs. Her father had a subarachnoid hemorrhage from a ruptured berry aneurysm. She is treated with prednisone, leading to marked and rapid improvement in her symptoms. Which of the following pathologic processes is most likely responsible for the patient's condition? A. granulomatous inflammation of the media B. trasmural inflammation with fibrinoid necrosis

A. granulomatous inflammation of the media Giant cell arteritis (GCA) is characterized by granulomatous inflammation of the media with intimal thickening and predominantly involves branches of the carotid artery, especially in the temporal artery. It is strongly associated with polymyalgia rheumatica; both conditions respond promptly to glucocorticoid therapy. Transmural inflammation of the arterial wall with fibrinoid necrosis is consistent with polyarteritis nodosa, an uncommon form of vasculitis typically occuring in young adults. It presents with intermittent episodes of a variety of manifestations including abdominal pain, peripheral neuropathy, renal insufficiency, and severe hypertension.

A 45-year-old man has a sore throat for the past 3 months with pain upon swallowing. He has no history of smoking, drinks occassionally, and does not use illicit drugs. On physical exam, his right tonsil is enlarged with small area of ulceration. Biopsy of the tonsil is shown: https://i.imgur.com/F0uf8Ey.png. Which of the following most likely played the most significant role in pathogenesis of this patient's condition? A. human papilloma virus B. herpes simplex virus

A. human papilloma virus Most oropharyngeal carcinomas (tongue, base of tongue) are caused by HPV infection. HPV-16 and HPV-18 are particularly likely to cause malignant transformation. The biopsy reveals malignant cells with atypical mitotic figures and keratin pearls consistent with head and neck mucosal squamous cell carcinoma. HSV-1 causes latent infection in sensory nerves, and it is associated with vesicular lesions of the orolabial mucosa, not tonsillar ulcers, and does not cause HNSCC.

A 42-year-old man with a long history of type 1 diabetes mellitus have frequent involuntary loss of urine. For several months, he has had difficulty starting and maintaining a urinary stream. In the last 3 weeks, he had 2 episodes of nocturnal enuresis, multiple daytime episodes of uncontrolled voiding without any sensation of a full bladder. His other medical problems include CKD and gastroparesis. What additional finding will most likely be found? A. increased postvoid residual volume B. loss of sensation in perineal area

A. increased postvoid residual volume Diabetic autoimmune neuropathy is common in type 1 diabetics and can cause overflow incontinence due to inability to sense a full bladder and incomplete emptying. Postvoid residual (PVR) testing with US or catheterization can confirm inadequate bladder emptying. Loss of sensation in the perineal area (saddle anesthesia) can indicate cauda equina syndrome, which is commonly due to epidural cord compression from a malignancy. Patients usually develop urinary retention late in the course of the disease, usually associated with fecal incontinence.

A 66-year-old man has a history of heart failure with reduced ejection fraction and persistent dyspnea despite taking maximal dosage of his heart failure medications. He has had no lightheadedness or chest pain. His medication regiment is modified with addition of a combination pill that inhibits nephrilysin and blocks angiotensin II receptors. What is the most likely effect of this medication? A. increased urinary sodium excretion B. decreased renal free-water reabsorption

A. increased urinary sodium excretion Nephrilysin is responsible for the breakdown of natriuretic peptides and angiotensin II; therefore, inhibition of nephrilysin increases the activity of these peptides. For treatment of heart failure, nephrilysin inhibition is combined with angiotensin II receptor blockade to optimize the positive effects of the natriuretic peptides (Vasodilation, diuresis) while blocking the negative effects of angiotensin II (vasoconstriction, fluid retention). Increased levels of ANP and BNP would stimulate the excretion of both salt and water. Decreased reabsorption of free water (water without solute) can be achieved with medications that block vasopressin-2 receptors (tolvaptan) on the renal collecting tubule.

A 44-year-old man has progressive dyspnea for several years. Physical exam shows a prolonged expiratory phase without wheezes or rhonchi. CT scan demonstrates bilateral lower lobe-predominant emphysema. Further testing reveals a protease inhibitor deficiency leading to increased elastin fiber breakdown. Elastin fibers within alveolar walls normally allow the lung to stretch during active inspiration and recoil during passive expiration. What contributes to this property of elastin? A. interchain cross-links involving lysine B. heavy post-translational hydroxylation

A. interchain cross-links involving lysine The rubber-like properties of elastin are due to high content of nonpolar (hydrophobic) amino acids and extensive cross-linking between elastin monomers facilitated by lysyl oxidase. Patients with alpha-1 antitrypsin deficiency can develop early-onset, lower lobe-predominant emphysema due to excessive alveolar elastin degradation. Post-translational hydroxylation is important in procollagen molecules.

A 25-year-old man has acute-onset shortness of breath. He has a history of cystic fibrosis and multiple hospitalizations for recurrent pneumonia. He has a frequent productive cough at baseline. BP is 80/50, pulse is 110, RR is 24. Exam shows mild cyanosis and subQ crepitus. Breath sounds are decreased on the left. Which of the following is responsbile for the patient's acute symptoms? A. loss of intrapleural negative pressure B. alveolar consolidation due to inflammatory exudate

A. loss of intrapleural negative pressure Spontaneous pneumothorax is common in CF and involves alveolar rupture, leading to loss of intrapleural negative pressure. Patients have sudden-onset shortness of breath, unliterally decreased breath sounds, and, sometimes, subQ crepitus. Hypotension and tachycardia suggest the development of tension pathophysiology. Alveolar consolidation with inflammatory exudate occurs with pneumonia, which can cause hypotension (septic shock). However, an increased breath sounds are expected with pneumonia, and it would not explain subQ crepitus.

A 45-year-old man is hospitalized for suicical ideation. Over the past month, he has been increasingly depressed, withdrawn, and physically restless. He has increasing difficulty concentrating at his job as a computer programmmer. Heis appetite has been poor and he has had difficulty falling and staying asleep. He stopped going to work last week and refused to leave the house until his hospitalization. He has not medical or psychiatric history. He has lost 10 lbs. On mental exan, he appears depressed and anxious. He reports he is "responsible for all the evil in the world" and has heard a voice for the past week telling him that he does not deserve to live. What is the most likely diagnosis? A. major depressive disorder with psychotic features B. schizophreniform disorder

A. major depressive disorder with psychotic features MDD with psychotic features is a severe subtype of unipolar MDD characterized by symptoms meeting the criteria for MDD and the presence of delusions and/or hallucinations. Requires 2+ weeks of depressed mood, impaired concentration, decreased appetite/weight loss, sleep disturbance, and suical ideation for MDD + psychotic symptoms for this subtype. Schizophreniform disorder is psychotic symptoms in the absence of a mood disorder, and a mood disorder is certainly present here.

A 10-year-old girl has short stature, low hairline, short and wide neck, a broad chest, and widely spaced nipples. She has not menstruated and has no breast buds, axillary, or pubic hair. Which of the following is the most likely underlying mechanism for this condition? A. meiotic nondisjunction B. uniparental disomy

A. meiotic nondisjunction Patients with Turner Syndrome may have karyotype 45,X (complete monosomy), 45 X/46,XX (mosaicism), or 46,XX (with partial deletion of one X chromosome). Complete monosomy X usually results from meiotic nondisjunction during gametogenesis. Uniparental disomy occurs when an individual inherits 2 copies of a chromosome from one parent and no copies of the chromosome from the other parent. For instance, although most often due to chromosomal deletions, uniparental disomy can also cause Angelman and Prader-Willi syndromes due to loss of expression of maternal/paternal imprinted components of a critical region of chromosome 15.

A 68-year-old man has a weak urinary stream, hesitancy, and straining on micturition. These symptoms have been present for the past 2 years but have gradullay become more severe. The patient started noticing continuous urine leakage, nocturia, frequent urination, and urgency. The patient's kidneys are most likely to demonstrate: A. parenchymal pressure atrophy B. glomerular sclerosis and hyalinosis

A. parenchymal pressure atrophy BPH leads to progressive bladder outlet obstruction. Over time, increased urinary pressures can cause hydronephrosis and renal parenchymal atrophy with scarring. This can progress to chronic kidney disease. Glomerular sclerosis and hyalinosis are typical of diabetic nephrosclerosis and focal segmental glomerulosclerosis (FSGS). Diabetes can cause autonomic neuropathy with overflow incontinence, but it is rare and typically occurs in poorly controlled chronic diabetes. FSGS does not cause lower urinary tract symptoms

A 15-month-old boy is brought to the ED with fever and foot swelling. He developed foot swelling of the right foot and fever 4 days ago. today, swelling of the left foot also developed. Hx is significant for perianal abscess at 9 months of age. Imaging reveals osteomyelitis affecting the metatarsal bones of both feet. IV antimicrobials are administered but the patient still has fever and bilateral foot swelling 2 weeks later. Lab workup shows persistent neutrophilic leukocytosis. Bone biopsy grows Staph aureus. Eval for diagnosis is initiated via use of a rhodamine derivative. The patient's peripheral blood is stimulated with phorbol myristate acetate (PMA) and compared with a control. Results of the test are shown here: https://i.imgur.com/dKEFcK8.png. What is most likely impaired in this patient? A. phagocytic metabolism B. opsonization

A. phagocytic metabolism Chronic granulomatous disease (CGD) is due to defective NADPH oxidase and is diagnosed by dihydrorhodamine testing, which measures neutrophil respiratory burst. When NADPH oxidase is stimulated, dihydrorhodamine is normally oxidized and fluoresces green (increased fluorescence intensity); it remains colorless (no oxidation) in patients with CGD. Opsonization is impaired in patients lacking IgG (X-linked agammaglobulinemia). The typical presentation involves recurrent sinopulmonary infections by encapsulated bacteria, not S. aureus. Moreover, the patient's DHR test is diagnostic of CGD.

A 26-year-old man comes to the ED with fever, right flank pain, and difficulty walking for the past 3 days. A week ago, he took cephalexin for bacterial folliculitis. He was diagnosed with type I diabetes 10 years ago and takes subQ insulin. Temp is 102 F. During medical interview, the patient lies supine on the exam table with his right hip and knee flexed and the limb externally rotated. On physical exam, he resists passive extension at the right hip due to worsening of the pain. This patient's pathological process most likely involves which of the following muscles? A. psoas major B. obturator externus

A. psoas major Psoas abscess presents with fever, back or flank pain, an inguinal mass, and difficulty walking. Inflammation of the psoas muscle leads to pain with extension at the hip (psoas sign). Psoas abscess can develop due to hematogenous or lymphatic seeding from a distant site, particularly in patients with diabetes mellitus, IV drug use, and immunosuppression (HIV infection). Obturator rotates the thigh.

A 32-year-old man has 1 week of progressive shortness of breath and cough. He reports sharp right-sided chest pain that is worsened with deep inspiration. Exam reveals decreased tactile fremitus, dullness on percussion, and decreased breath sounds over the right lower lung. Xray shows consolidation of the right lower lobe and a right-sided effusion. A thoracentesis is performed, during which the needle is inserted alon the upper border of the 10th rib at the right midaxillary line. Which structure is most at risk of being injured as a result? A. right hepatic lobe B. intercostal nerve

A. right hepatic lobe Thoracentesis should be performed below the 6th rib in the midclavicular line, the 8th rib along the midaxillary line, or the 10th rib along the paravertebral line in order to minimize the risk of lung injury. Insertion of a needle lower than 9th rib increases the risk of penetrating abdominal structures. The needle should be inserted along the UPPER border of the rib to prevent injury to the intercostal vessels. The intercostal vein, artery, and nerve lie in the subcostal groove along the lower border of the rib. Thoracentesis should be performed just above the upper border of the rib to prevent injury to the intercostal vessels

A 43-year-old woman has ongoing abdominal pain, general weakness, decreased appetite, and dizziness. She says her pain is ruining her life, and she is worried the previous physicians may have missed something. She has been hospitalized 3 times last year with similar symptoms. No etiology for the pain has been identified despite extensive workups, several CT scans, laparotomy, and lab tests all appearing normal. She describes chronic abdominal pain since adolescence and is concerned as the nonprescription analgesics she takes are ineffective. What is the most appropriate next step? A. schedule regular outpatient office visits B. refer for psychiatric treatment

A. schedule regular outpatient office visits Somatic symptom disorder is best managed with regularly scheduled medical visits that are not contingent on having active symptoms. Unnecessary diagnostic testing and specialist referrals should be avoided.

A 67-year-old man with nonischemic cardiomyopathy was recently hospitalized for acute decompensated heart failure. His symptoms have improved with multidrug treatment, but he has persistent shortness of breath on mild exertion. He has a history of hypertension and hypercholesterolemia. There is a 3rd heart sound on auscultation and mild lower extremity pitting edema. A recend echocardiogram shows a left ventricular ejection fraction of 30%. Which of the following diuretics will most likely improve survival if added to this patient's regiment? A. spironolactone B. furosemide

A. spironolactone Mineralocorticoid receptor antagonists (sprionolactone, eplerenone) improve survival in patients with CHF and reduced LVEF. They should not be used in patients with hyperkalemia or renal failure. Furosemide, a loop diuretic, is frequently used for treatment of pulmonary congestion and fluid retention in heart failure patients. Although loop diuretics improve symptoms significantly, they do not provide survival benefit (ie, improved morbidity but not mortality) in these patients.

A 34-year-old female presents with a small bluish lesion under the nail of her right index finger. It is extremely tender to touch. If the lesion is a tumor, its cell of origin is most likely to have which of the following functions: A. thermoregulation B. light touch

A. thermoregulation A benign glomus tumor (glomangioma) can produce a very tender, small (few mm in diameter), red-blue lesion under the nail bed. This type of tumor originates from the modified smooth muscle cells that control the thermoregulatory functions of dermal glomus bodies.

A 26-year-old man has severe headaches that are increasing in frequency. The patient describes episodic, right-sided hemicranial throbbing or pulsating headaches accompanied by nausea and photophobia. These headaches began in high school, but they have become more frequent and are affecting his work. The patient takes NSAIDs and sumatriptan for pain and metoclopramide for nausea. He has no allergies, normal vitals, and a brain imaging study was normal. What is the most helpful pharmacotherapy to prevent recurrences for this patient's headaches? A. topiramate B. ergotamine

A. topiramate Migraine is an episodic disorder characterized by severe, often unilateral headache, commonly associated with nausea and sensitivity to light and sound. Treatment is divided into abortive therapy (mild analgesics, triptans, antiemetics, ergotamines) for acute symptoms and preventative therapy (beta blockers, TCAs, anticonvulsants like topiramate, valproate) to reduce the frequency of headaches. Ergotamine has antagonist and agonist activities on multiple neurotransmitters. It is used for migraine abortion, not prevention. Topiramate is an anticonvulsant that reduces freq.

A 62-year-old man underwent mitral valve replacement 1 month ago and now is being evaluated for low-grade fevers, malaise, and dyspnea. Blood cultures grow gram-positive cocci in clusters that are catalase-positive and coagulase-negative. Initial antibiotic treatment should include which of the following? A. vancomycin B. ciprofloxacin

A. vancomycin Initial empiric treatment of coagulase-negative staphylococcal infection should include vancomycin due to widespread methicillin resistance, especially in nosocomial infections. If susceptibility results indicate a methicillin-susceptible isolate, vancomycin can be switched to nafcillin or oxacillin. (BTW low grade fevers associated with mitral valve replacement screams coagulase-negative staphylococci like Staph epidermidis). Ciprofloxacin resistance appears to accompany methicillin resistance in staphylococci due to the simultaneous presence of multiple antibiotic resistance genes.

A 53-year-old woman has an aching pain in her legs. She has a history of migraine and untreated chronic hepatitis C. She smokes a pack of cigarettes daily but does not use alcohol or recreational drugs. Physical exam findings are shown here: https://i.imgur.com/X1sOhZO.png. An abnormality affecting which of the following is the most likely cause? A. venous valves B. plasma oncotic pressure

A. venous valves Chronically elevated venous pressure in the lower extremities can lead to incompetent venous valves and venous dilation (varicose veins). Venous congestion and tissue ischemia can result in venous stasis dermatitis. Decreased plasma oncotic pressure results primarily from low plasma protein content (nephrotic syndrome, cirrhosis/liver failure) and results in generalized edema. Cirrhosis with portal hypertension can cause esophageal and anorectal varices as collateral channels from the portal to systemic circulation. However, lower extremity veins do not communicate with the portal circulation.

A 60-year-old woman comes to the ED due to shortness of breath and productive cough. She has a 30-pack-year smoking history. O2 satuation is 88%. On physical exam she appears uncomfortable and uses her accessory respiratory muscles to breathe. Expiratory wheezes are heard throughout the lungs. The patient is started on high-flow oxygen supplementation. Shortly afterwards, she becomes increasingly lethargic and confused. Her clinical decline is most likely attributable to an increase in which of the following parameters? A. ventilation-perfusion mismatch B. pulmonary vascular resistance

A. ventilation-perfusion mismatch Supplemental oxygen administration in patients with COPD can lead to increased CO2 retention (oxygen-induced hypercapnia), resulting in confusion and depressed consciousness. The major cause is REVERSAL of hypoxic pulmonary vasoconstriction, leading to an increase in physiologic dead space as blood is shunted away from well-ventilated alveoli. Hypoxia causes vasoconstriction of the pulmonary arterioles, acting to shunt blood towards alveoli with highest ventilation, minimizing physiologic dead space. Providing high-concentration supplemental oxygen allows lung regions with relatively poor ventilation to have higher oxygen levels, REVERSING pulmonary vasoconstriction. The redistribution of blood flow away from well-ventilated alveoli leads to an increase in physiologic dead space (well-ventilated alveoli are less perfused) with a corresponding reduction in CO2 excretion.

A 33-year-old man has fatigue and decreased exercise tolerance over the past 6 months. He was diagnosed with infective endocarditis in the setting of IV drug use a year ago and completed a course of antibiotic therapy. He has not used drugs since then. A cardiac murmur is heard on auscultation. Both carotid arteries demonstrate a rapid rise and fall of the arterial pulse. Partial compression of the femoral arteries by the stethoscope elicits a systolic-diastolic bruit. Which of the following findings is most likely to be observed during cardiac catheterization of this patient? A. Aortic pressure 110/75 and left ventricular pressure 160/20 B. Aortic pressure 160/60 and left ventricular pressure 160/20

B. Aortic pressure 160/60 and left ventricular pressure 160/20 Aortic regurgitation causes a rapid fall in aortic pressure during diastole with an increase in left ventricular end-diastolic volume and a compensatory increase in stroke volume. These hemodynamic changes create a characteristic pressure change, including reduced aortic diastolic pressure, increased aortic systolic pressure, and increased left ventricular diastolic and systolic pressures.

A 13-year-old boy with cystic fibrosis has a persistent cough over the past 2 weeks despite treatment with several oral antibiotics. Temp is 98.1. Pulmonary exam reveals diffuse wheezing. PFT is consistent with exacerbation of the underlying disease. The patient is admitted for broad-spectrum IV antibiotics and aggressive respiratory therapy. Induced sputum is collected for cultures. In addition to blood, chocolate, and MacConkey agar, this patient's sputum should be cultured on which of the following media? A. Bordetella pertussis-selective agar B. Burkholderia cepacia-selective agar

B. Burkholderia cepacia-selective agar Patients with CF are at risk for pulmonary exacerbation due to Staph aureus, Pseudomonas aeruginosa, nontypeable Haemophilus influenzae, and Burkholderia cepacia complex. The other answer does not commonly cause CF exacerbation.

A 44-year-old man with progressive dyspnea is diagnosed with dilated cardiomyopathy. Despite optimal therapy, his disease and symptoms progress. Microelectrodes are placed into cardiac muscle cells and a rapid decrease in cytoplasmic calcium level immediately preceding relaxation. Which of the following proteins is most responsible for the observed change in electrolyte levels? A. ryanodine receptors B. Na/Ca exchanger

B. Na/Ca exchanger Calcium efflux from cardiac cells prior to relaxation is primarily mediated via an Na/Ca exchange pump and sarcoplasmic reticulum Ca-ATPase pump. The INITIAL calcium influx by L-type voltage-dependent calcium channels is sensed by ryanodine receptors in the sarcoplasmic reticulum, triggering the release of FURTHER Ca into the cytoplasm, increasing intracellular calciulm concentration 100-fold.

A 55-year-old woman has persistent morning stiffness. She was diagnosed with rheumatoid arthritis 4 weeks ago and was prescribed methotrexate. She takes the maximum tolerated dose, along with folic acid and as-needed naproxen. On exam, swelling, tenderness, and pain on range of motion are found at the metacarpophalangeal and proximal interphalangeal joints and wrists bilaterally. Treatment with etanercept is considered. Which of the following tests should be performed before beginning treatment with this agent? A. pulmonary function test B. TB skin test

B. TB skin test Tumor necrosis factor-alpha (TNF-a) inhibitors impair cell-mediated immunity. All patients beginning treatment with TNF-a inhibitors should be evaluated for latent TB, because TNF-a is necessary for effective sequestration of mycobacteria within granulomas. PFTs are important for medications that may cause fibrosis or lung damage, such as amiodarone or methotrexate.

A 56-year-old woman has a pigmented lesion on her back that has enlarged since she noticed it a year ago. She is a forest service ranger for 30 years. On exam, there is a large irregular pigmented lesion on the upper back, as shown here: https://i.imgur.com/OrblHQ1.png. There are scattered patches of erythema and telangiectasias. Excisional biopsy of the lesion shows malignant melanoma. What is associated with the highest risk of metastasis? A. prominent cellular atypia B. active vertical growth

B. active vertical growth Melanoma often has an early horizontal growth phase with low metastatic potential followed by a nodular, vertical growth phase with a significantly increased risk of metastasis. Depth of invasion (Breslow thickness) is the most important prognostic indicator in malginant melanoma

A 64-year-old man has flank discomfort and red urine. He has a history of hypertension and type II diabetes mellitus. 3 months ago, he had an ischemic stroke and now has mild, residual, right-sided weakness. Serum creatinine is 0.9 and serum LDH is elevated. Urine microscopy shows many RBCs. CT scan is shown: https://i.imgur.com/UoiFSBg.png. What is the most likely cause of this patient's symptoms? A. focal segmental glomerulosclerosiis B. atrial fibrillation

B. atrial fibrillation Renal infarction presents with flank pain, hematuria, elevated LDH, and a wedge-shaped kidney lesion on CT. The most common cause of renal infarction is systemic thromboembolism, often due to thrombus formation during Afib. The brain and kidneys are more likely than other organs to suffer embolic infarctions because they are perfused at a higher rate. Focal segmental glomerulosclerosis manifests as nephrotic syndrome. Urinary loss of certain natural anticoagulants (AT III) leads to a hypercoagulable state. However, this rarely causes systemic embolization and patients usually have elevated Cr.

An employer would like to negotiate a contract in which a medical group provides care to all of the company's employees in exchange for a set monthly fee per employee. What payment method best describes this type of health care financing arrangement? A. point-of-service B. capitation

B. capitation Capitation is an arrangement in which a payor pays a fixed, predetermined fee to provide all the services required by a patient. Payors may negotiate a capitated contract with an insurance company that then pays the providers, or a large medical group may negotiate directly with the payor. Point-of-service plans require patients to have a primary care provider and obtain referrals for specialty consultations. They differ from HMO plans in that they allow patients to see providers outside the network, albeit at higher out-of-pocket costs (copays and deductibles)

A 58-year-old man has upper abdominal pain, fatigue, and anorexia. He says these symptoms began a month ago and seem to be getting worse. He has a history of cirrhosis due to chronic HepC infection. Physical exam shows cachexia and worsening ascites. Imaging studies reveal a large mass in the right hepatic lobe consistent with unresectable hepatocellular cancer. He is scheduled to undergo percutaneous embolization of the artery supplying the tumor. Contrast material administration into which of the following structures is most likely to visually enhance the artery prior to embolization? A. superior mesenteric artery B. celiac trunk

B. celiac trunk The celiac trunk is the first main branch of the abdominal aorta; it provides blood to the spleen, stomach, liver, abdominal esophagus, and parts of the duodenum and pancreas. The proper hepatic artery branches off the common hepatic artery from the celiac trunk and provides arterial blood supply to the liver. The superior mesenteric artery branches off the abdominal aorta inferior to the origin of the celiac trunk. It supplies blood to the part of the pancreas and the intestine from the lower part of the duodenum to the first two-thirds of the transverse colon.

An 18-year-old man comes to the ED with a 3-day history of fever, myalgias, and profound fatigue. He was feeling fine prior to symptom onset. He has a history of intermittent asthma for which he uses an inhaler. He is a senior in high school and plays on the football team. Temp is 100.4 and BMI is 22.1. On physical exam, the patient is found to have swollen lymph nodes in his neck and behind his ears. The tip of the spleen is palpable 1 cm below the left costal margin. Lab tests reveal elevated leukocytes and 70% lymphocytes. What is most likely to be elicted on further history? A. recent tick bite B. contact with saliva

B. contact with saliva Typical clinicl and lab features of EBV mononucleosis are fever, pharyngitis, lymphadenopathy, splenomegaly, and atypical lymphocytosis. EBV is generally transmitted from an asymptomatic virus shedder to a susceptible individual through saliva transfer.

23-year old man is dehydrated after a desert journet. Which of the folllowing nephron segments is likely to be most hypoosmotic? A. collecting duct B. distal convoluted tubule

B. distal convoluted tubule HYPOOSMOTIC = DILUTE. Dehydration (hypotension, high plasma osmolarity) increases circulating levels of ADH, increasing water permeability of the collecting duct and allowing production of HIGHLY CONCENTRATED, HYPEROSMOTIC urine. In the dehydrated state, tubular fluid osmolarity will be lowest at the begninning of the distal convoluted tubule.

A 46-year-old woman presents with a painful rash involving her groin and legs that has worsened over the last 2 weeks. She was diagnosed with diabetes mellitus 6 months ago and has no other medical problems other than occassional loose stools. Exam shows coalescing erythematous plaques with crusting and scaling at the borders and central areas of brownish indurations. Biopsy of the lesion reveals superficial necrolysis. What lab abnormality is most likely to be found? A. elevated vasoactive intestinal polypeptide B. elevated glucagon

B. elevated glucagon Glucagonoma presents with hyperglycemia (often as newly diagnosed diabetes mellitus) and necrolytic migratory erythema (blistering erythematous plaques with central clearing) affecting the groin, face, and extremities. The diagnosis is made by detecting elevated glucagon levels. High levels of VIP are seen in patients with VIPoma, which typically presents with intractable diarrhea, hypokalemia, and achlorhydria. Patients usually are hypotensive due to dehydration and vasodilatory effects of VIP.

a 40-year-old man has lesions over his thigh that he noticed 2 weeks ago. They are neither painful nor itchy. He has a history of HIV infection and has not been compliant with therapy. On physical exam he is mildly cachectic. Vital signs are normal. There are 2 elliptical, raised, nontender, violaceous lesions on his right thigh. No other skin or mucous membrane lesions are present. There is no lymphadenopathy. CD4 count is 100. The patient's skin condition is most likely originated from which of the following cells? A. basal keratinocytes B. endothelial cells

B. endothelial cells Kaposi sarcoma often presents as multiple red, purple, or brown lesions on the lower extremities in patients with HIV infection. It is a vascular tumor caused by HSV-8 infection of endothelial cells. Basal cell carcinoma arises from basal keratinocytes, typically due to UV radiation. Lesions are commonly pink or skin-colored (vs violaceous), pearly or translucent, and located in sun-exposed areas such as the face (not the thigh).

A 37-year-old woman has exertional shortness of breath. She has had progressive dyspnea for the last year and the last 2 months she has been unable to walk half a block without stopping to rest. She also describes lightheadedness during exertion. BMI is 23. Physical exam reveals 2/6 holosystolic murmur at the lower sternal border which increases with inspiration. Further evaluation indicates no evidence of obstructive or interstitial lung disease or venous thromboembolism. Cardiac catheterization reveals elevated mean pulmonary artery pressure with normal PCWP. Pharmacotherapy blocking the effects of which of the following will benefit this patient? A. Angiotenin II B. endothelin

B. endothelin Pulmonary hypertension can occur due to a primary change in the pulmonary arteries (pulmonary arterial hypertension) or it can be secondary to a separate disease process such as left-sided heart failure (ruled out by normal PCWP). Endothelin receptor antagonists (bosentan, ambrisentan) lower pulmonary arterial pressure and improve dyspnea in patients with pulmonary arterial hypertension. Ang II receptor blockers like losartan help prevent cardiac remodeling in patients with left ventricular systolic dysfunction. These drugs can be used to treat pulmonary hypertension that develop secondary to left-sided heart failure, but they are not used for PAH as a direct effect on pulmonary vascular remodeling has not been established.

What is the pathway of pleural fluid turnover? A. entry by bronchial microvessels and exit by pulmonary parenchymal lymphatics B. entry by intercostal microvessels and exit by parietal pleural lymphatics

B. entry by intercostal microvessels and exit by parietal pleural lymphatics Pleural fluid normally enters the pleural space via filtration from the systemic circulation, primarily from the intercostal microvessels of the parietal pleura, and exits the pleural space via stomata in the parietal pleura that drain into the parietal pleura lymphatics. Pulmonary parenchyma lymphatics do not contribute to the uptake of pleural fluid.

A 32-year-old woman is recruited for a research study. During the evaluation, increases in HR and CO are observed with normal partial pressures of arterial oxygen and CO2. These changes are most likely part of an integrated response to which of the following? A. carotid sinus massage B. exercise

B. exercise The cardiorespiratory response to exercise includes increased HR, CO, and RR in order to balance the increased total tissue oxygen consumption and CO2 production. These coordinated adaptations result in relatively constant arterial blood gas values whereas venous oxygen is decreased and venous CO2 is increased. Carotid sinus massage causes reflex PSNS stimulation of SA node, atrial myocytes, and AV node, resulting in transeint decrease in HR and CO.

A 34-year-old woman has dysuria and a history of recurrent UTIs. Gram negative bacteria form pink colonies on lactose-containing MacConkey agar. Several days later, a second urine sample grows white colonies on the same medium. Genetic analysis shows that the more recent isolates have a single nucleotide deletion within the lac operon DNA sequence. The genomic change is most consistent with which of the following? A. missense mutation B. frameshift mutation

B. frameshift mutation A frameshift mutation occurs with the deletion/addition of a number of bases not divisible by 3 in the coding region of a gene. Frameshift mutations alter the reading frame of the genetic code, dramatically changing the protein structure and often resulting in the formation of a premature stop codon. Missense mutations are single base mutations that result in the placement of an incorrect amino acid in a protein sequence. These mutations occur within the coding region of a gene and do not alter protein length.

A 25-year-old woman has 2 days of abdominal pain and bloody diarrhea. A few days ago she attended a church barbeque, but she is unsure if others have similar symptoms. There is abdominal tenderness with no rebound or guarding. Guaiac-positive bloody stools are detected on rectal exam. Stool cultures reveal an E. coli strain that does not produce glucuronidase and does not ferment sorbitol, consistent with serotype O157:H7. Which of the following mechanisms best describes the action of the toxin specific to these bacteria? A. stimulation of chloride secretion B. inactivation of 60S ribosomal subunits

B. inactivation of 60S ribosomal subunits Shiga toxin-producing E. coli O157:H7 elaborates Shiga toxin (STEC: Shiga Toxin-producing E. coli), which inactivates 60S ribosomal subunits in colonic mucosal cells, leading to the inhibition of protein synthesis and apoptosis; clinical manifestations include bloody diarrhea. Hematogenous spread of Shiga toxin to renal endothelial cells can also lead to hemolytic uremic syndrome. Unlike STEC, enterotoxigenic E. coli produces heat-labile toxin (LT) and heat-stable toxin (ST). LT increases intracellular cAMP in intestinal mucosal cells, which leads to the decreased absorption and increased secretion of sodium, chloride, and water. ST increases intracellular cGMP, also contributing to diarrhea and electrolyte loss. Watery, not bloody, diarrhea occurs.

A 45-year-old woman diagnosed with Crohn disease 3 years ago has recurrent RUQ abdominal pain. The pain is 5/10, dull, and occassionally radiates to the right shoulder. It typically occurs after eating fatty meals and is associated with nausea. The patient has had no fever, vomiting, melena, or hematochezia. She takes infliximab for Crohn disease and atorvastatin for hyperlipidemia. Abdominal US reveals multiple mobile calculi within the gallbladder. Which of the following processes is most likely responsible for the development of gallstones in this patient? A. gallbladder hypomotility B. increased bile acid wasting

B. increased bile acid wasting Patients with Crohn disease affecting the terminal ileum (most common site of involvement) are prone to developing gallstones. Decreased bile acid reabsorption by the inflamed terminal ileum promotes cholesterol supersaturation of the bile, resulting in gallstone formation. Gallbladder hypomotility may contribute to cholesterol gallstone formation in patients with spinal cord injury or those receiving total parenteral nutrition; however, this is not observed in Crohn's disease

A 24-year-old man comes to the ED with severe nausea and vomiting 2 hours after eating chicken salad that has been sitting in the sun. He is afebrile with normal vitals. No abdominal tenderness is present and bowel sounds are normal. Blood cell counts and serum chemistry are normal. Symptoms improve significantly within several hours with supportive care. What is the most likely cause of this patient's symptoms? A. microbial attachment to the intestinal surface B. ingestion of preformed enterotoxin

B. ingestion of preformed enterotoxin Staphylococcal (Staph aureus) food poisoning is mediated by the ingestion of a preformed, heat-stable entercotoxin that induces rapid-onset (<6 hours) nausea and vomiting. Most causes arise due to improper food handling and storage. Common culprit foods include eggs, dairy products, and mayonnaise-based salads. Enteropathic E. coli attaches to enterocytes to cause watery diarrhea, but this strain does not produce toxins.

A 29-year-old woman has unintentional weight loss, fatigue, and sweating. She has no neck pain. Two months ago she delivered a healthy boy with no perinatal complications. She did not breastfeed the infant. Neck exam shows mild, diffuse enlargement of the thyroid gland with no tenderness. The remainder of the physical/pelvic exam is normal. Lab exam reveals low serum TSH, elevated serum T4, and elevated serum T3. Thyrotropin receptor antibodies are negative. Which of the following is the most likely cause of the patient's symptoms? A. tumor production of beta-hCG B. lymphocytic inflammation of the thyroid gland

B. lymphocytic inflammation of the thyroid gland Postpartum thyroiditis occurs within 12 months of pregnancy and is characterized by autoimmune destruction of thyroid follicles. It typically begins with a hyperthyroid phase due to the release of preformed thyroid hormone, followed by a hypothyroid phase due to depletion of thyroid hormones and an eventual return to a euthyroid state. Histologic inspection demonstrates lymphocytic infiltration with the formation of germinal centers, which is similar to chronic lymphocytic (Hashimoto) thyroiditis. Gestational trophoblastic neoplastia refers to a group of hCG-secreting tumors that most commonly occur in association with molar pregnancy (uncommon following live birth). Most patients have associated symptoms such as amenorrhea or abnormal uterine bleeding, and exam often shows uterine enlargement.

A 6-year-old girl is brought to the ED after falling from a chair onto her outstretched right arm. Physical exam shows swelling of the right elbow and tenderness to palpation over the distal humerus. Radial pulse is slightly diminished in the right upper extremity compared to the left. The patient cannot cooperate with neurological exam of the right hand due to significant pain. A peripheral IV line is placed, and analgesics are administered for pain control. Xray reveals a supracondylar humeral fracture with anteromedial displacement of the proximal fracture fragment. Which of the following structures is most likely to be injured in this patient? A. ulnar nerve B. median nerve

B. median nerve Supracondylar humeral fractures commonly occur after hyperextension of the elbow as a result of a fall onto an outstretched arm. The median nerve and brachial artery run along the anteromedial aspect of the elbow and are the structures most likely to be injured with anteromedial displacement of the proximal fracture fragment. The ulnar nerve runs posterior to the medial epicondyle afnd can be injured with hyperflexion injuries (falling onto flexed elbow), resulting in posterior displacement of the proximal humerus or fracture of the medial epicondyle. Ulnar nerve injury at the elbow often presents with sensory loss over the medial hand, fifth digit, and half of the fourth digit, as well as weakness on finger abduction/adduction and flexion of the wrist and fourth/fifth digits. Helpful pics: https://imgur.com/a/79z9aGi

A 1-hour-girl born to a 40-year-old woman is brought to the nursery for eval. Physical exam shows mid-face hypoplasia with a flat nasal bridge, up-slanting palpebral fissures, a small mouth, and a single palmar crease bilaterally. Cardiac auscultation reveals a blowing holosystolic murmur heard best along the sternal border. Which of the following abnormalities is most likely to be present in this patient? A. uniparental disomy B. mosaicism

B. mosaicism Common findings in Down syndrome include cognitive impairment, facial dysmorphism, and cardiac defects; 95% of cases are caused by the presence of an extra chromosome 21 (trisomy) resulting from nondisjunction. Unbalanced Robertsonian translocations or mosaicism are less common causes. Aberrant imprinting occurs with uniparental disomy, when a person receives 2 copies of a chromosome from the same parents and no copy from the other parent. This is the case in Prader-Willi and Angelman syndromes.

A 72-year-old man has a 1-week history of progressive confusion and lethargy. He has had a persistent cought for the past several weeks with 2 episodes of blood in the sputum. He has hyperlipidemia, well-controlled hypertension, and a 48-pack-year smoking history. Mucous membranes are moist. Lung auscultation reveals wheezing in the left lung. Serum sodium is 123 (low). Chest Xray reveals a mass in the upper left lobe. A bronchoscopy biopsy reveals an aggressive lung cancer. Which of the following is most likely to be present on further analysis of the patient's biopsy sample? A. KRAS mutation B. neural cell adhesion molecule

B. neural cell adhesion molecule Small cell carcinoma of the lung is the most aggressive type of lung cancer and is commonly associated with paraneoplastic syndromes (SIADH, Cushing syndrome). It is thought to have a neuroendocrine origin; tumor cells express neuroendocrine markers (neural cell adhesion molecule, chromogranin, synaptophysin) and contain neurosecretory granules in the cytoplasm. KRAS is an activating mutation present in up to 25% of lung adenocarcinomas and is associated with smoking. It is also frequently seen in colorectal and pancreatic cancerpatients

A 35-year-old expecting mother is taking methotrexate for rheumatoid arthritis. The patient's unborn child is at greatest risk for which of the following congenital defects? A. renal dysplasia B. neural tube defects

B. neural tube defects Methotrexate and other folic acid antagonists adversely affect rapidly dividing cells by limiting the production of precursors essential to DNA synthesis and repair. If used in the first trimester of pregnancy, these drugs can cause major congenital abnormalities (CV, neural tube defects). Renal dysplasia is a complication of ACE inhibitors in hypertensive mothers.

A 16-year-old girl comes to the ED with vaginal bleeding. Her last menstrual period was 12 weeks ago and she is sexually active without contraception use. She had a miscarriage last year that required dilation and curettage. Urine pregnancy test is positive. Transvaginal US shows intrauterine gestational sac without fetal cardiac activity, and dilation and curettage is performed. Pathology shows fetal tissue, focal trophoblastic hyperplasia, and some enlarged villi interspersed with normal villi. Which of the following is the most likely diagnosis? A. complete mole B. partial mole

B. partial mole A partial mole will have triploid karyotype (eg 69,XXX or XXY) and contain fetal tissue with some edematous villi with focal trophoblastic proliferation, and normal-appearing villi. Patients present with vaginal bleeding, and prior miscarriage is a risk factor. Complete moles have diffusely hyperplastic trophoblasts with diffusely enlarged villi and do not have any fetal tissue. Invasive moles are complete moles with myometrial invasion. Choriocarcinomas have diffusely anaplastic/necrotic trophoblasts with vascular invasion and absent villi.

Insulin increases glycogen synthesis in hepatocytes. Activation of which of the following molecules most likely promotes this metabolic effect? A. JAK B. protein phosphatase

B. protein phosphatase Insulin is an anabolic hormone that acts via receptor tyrosine kinase signaling to increase the synthesis of glycogen, proteins, FAs, and nucleic acids. Tyrosine kinase/phosphatidylinositol-3-kinase stimulation promotes glycogen synthesis by activating protein phosphatase, an enzyme that dephosphorylates (activates) glycoen synthase. JAK has tyrosine kinase activity; however, it is a cytoplasmic enzyme that is not part of a cell surface receptor (non-receptor tyrosine kinase).

A 19-year-old man has a painful subQ forearm nodule. Two weeks earlier, he received a laceration on his left forearm and had sutures placed. The lesion was biopsied: https://i.imgur.com/mY31vBj.png. What is most likely responsible for this patient's condition? A. excess granulation tissue formation B. reaction to a foreign body

B. reaction to a foreign body Granulomatous inflammation is a form of chronic inflammation characterized by aggregates of activated macrophages that assume an epithelioid appearance. Persistent granulomatous inflammation with subsequent fibrosis can can organ dysfunction, which is seen in a number of granulomatous diseases. Excess granulation tissue formation (hypergranulation) looks like this: https://i.imgur.com/7qkB8w2.png. It usually occurs in a wound left to heal by secondary intention. Light microscopy would show proliferation of fibroblasts, collagen deposition, inflammatory cells, and numerous capillaries.

A pharmaceutical researcher develops a new drug that affects bacterial protein synthesis. In an experiment, E. coli is exposed to the drug and is serially cultured in media containing tagged nucleotides and amino acids. It is found that the drug inhibits molecules that recognize the codon UAA in the bacterial mRNA fragment. Which molecule is the drug's target? A. charged tRNA B. releasing factor 4

B. releasing factor 4 Releasing factors recognize stop codons (UAA, UAG, IGA) and terminate protein synthesis. They facilitate release of the polypeptide chain from the ribosome and dissolution of the ribosome-mRNA complex. Charged tRNA delivers AA to the protein synthesis complex. The anticodon on a tRNA molecule recognizes the corresponding codon on mRNA, assuring proper AA sequencing

A 45-year-old quadriplegic man with suspected bacterial pneumonia is started on IV antibiotics. Over the next 24 hours, he develops progressive respiratory failure requiring mechanical ventilation. Prior to intubation, a SKM relaxant is administered and the patient subsequently goes into cardiac arrest. His attached cardiac monitor shows ventricular fibrillation. While being resuscitated, his serum potassium level is 9.5 mEq/L. Administration of which of the following agents is most likely responsible for this patient's condition? A. baclofen B. succinylcholine

B. succinylcholine Succinylcholine can cause significant potassium release and life-threatening arrythmias in patients at high risk for hyperkalemia, including those with burns, myopathies, crush injuries, and denervating injuries or disease

A 63-year-old postmenopausal woman has right-sided lower abdominal pain for the last 5 months. She has no changes in bowel or bladder habits, no increase in abdominal girth or bloating, and no early satiety. Physical exam shows soft abdomen with slight tenderness over the RLQ. Pelvic exam reveals the uterus is small and the right adnexa is enlarged. Transvaginal US shows a normal uterus. A unilteral 3.5-cm complex ovarian mass with cystic solid components and no free fluid in the pelvis. Surgery is planned to remove the right ovary, including the mass. To avoid excessive bleeding, which of the following structures should be ligated during the oophorectomy? A. round ligament of the uterus B. suspensory ligament of the ovary

B. suspensory ligament of the ovary The suspensory ligament of the ovary (infundibular ligament) contains the ovarian artery, ovarian vein, lymphatics, and nerves. The ovarian artery is the major blood supply to the ovary. Therefore, the suspensory ligament of the ovary must be ligated during an oophorectomy to prevent heavy bleeding. The round ligament of the uterus originates from the uterus (anteroinferior to the fallopian tube) and travels through the inguinal canal to attach to the labia majora. It contains the artery of Sampson, which is rarely a source of major bleeding during pelvic surgery.

A 10-year-old boy has abrupt onset fever and sore throat that began a day ago. He has had similar illnesses in the past treated with oral antibiotics. He has received all recommended vaccinations. Temp is 101F. Exam shows erythema of the posterior pharyngeal wall and enlarged tonsils covered with white exudate. There are no enlarged cervical lymph nodes. Throat swab is obtained. What is the most helpful in determining the need for antibiotic treatment here? A. serum titer assay for anti-streptolysin O antibodies B. swab immunoassay for strep group A antigens

B. swab immunoassay for strep group A antigens Group A strep should be suspected in those with acute-onset sore throat, exudative tonsillopharyngitis, and no evidence of viral symptoms (coryza, cough, conjunctivitis). In-office throat swab with rapid antigen detection testing (immunoassay for GAS antigens) can provide on-site confirmation for early treatment. Elevated antibody titers against strep O indicate recent GAS infection and can aid the diagnosis of GAS complications. However, antibodies against streptolysin take several weeks to form; therefore, they are not used to diagnose acute pharyngitis.

A 52-year-old woman has fatigue for the last 2 weeks. For the past 3 months, she has had significant weight loss without changing her diet. She reports increased sweating, hand tremor, and decreased sleep. Exam shows a diffuse, nontender enlargement of the thyroid gland. The eyeballs are protuberant. There is 1+ pitting edema in the ankles. Which of the following is most likely decreased in this patient? A. pulse pressure B. systemic vascular resistance

B. systemic vascular resistance Hyperthyroidism causes characteristic CV changes. Increased metabolic demand in the tissues and a direct vasodilatory effect of thyroid hormone lead to reduced systemic vascular resistance (and decreased diastolic blood pressure). A direct sympathetic-like effect of thyroid hormone on the myocardium leads to increased HR, contractility, and cardiac output. Increased stroke volume increases pulse pressure and systolic blood pressure.

A 48-year-old woman has a 6-month history of involuntary passage of urine while sneezing or coughing. Recently she has experienced urine leakage even with normal daily activity. She has no weakness, numbness, or fecal incontinence. She has hypertension and type 2 diabetes mellitus. She has had 4 uncomplicated vaginal deliveries. Supine BP is 126/82 and upright BP is 120/80. Pelvic exam shows normal vaginal rugae and physiologic discharge. A small amount of urine leaks from the urethra when the patient is asked to cough. Neurologic exam is normal. What is the most likely cause of this patient's condition? A. detrusor muscle overactivity B. urethral sphincter dysfunction

B. urethral sphincter dysfunction Stress incontinence, involuntary leakage of urine with increased intraabdominal pressure, can occur due to urethral sphincter dysfunction (decreased sphincter tone). Risk factors include multiple prior vaginal deliveries, which can injure the external urethral sphincter or pudendal nerve. Detrusor overactivity is the underlying mechanism of urge incontinence, which typically presents with a sudden urge to urinate followed by the immediate, involuntary loss of urine. This patient's incontinence episodes typically follow an abrupt increase in intraabdominal pressure (Coughing, sneezing), making urge incontinence less likely.

A 45-year-old man comes to the ED due to shortness of breath and fatigue that has progressed over the last 2 weeks. The patient was diagnosed with nonischemic cardiomyopathy 2 years ago and has not been adherent with physician visits and his medical regimen. Blood pressure is 106/58 mmHg with a regular normal pulse. The patient appears uncomfortable when lying flat. JVD is elevated. Bibasilar crackles are heard on auscultation and S3 is present. There is 1+ lower extremity pitting edema. Milrinone is initiated. What is the most likely response to this therapy? A. anriarrhythmic action B. vasodilation

B. vasodilation Milrinone is a phosphodiesterase-3 inhibitor that reduces the degradation of cyclic adenosine monophoshate to provide 2 beneficial effects for treating systolic heart failure. Calcium influx into cardiomyocytes is increased, which increases cardiac contractility. In addition, calcium-myosin light chain kinase interaction is reduced, which causes vasodilation and reduces cardiac preload and afterload. Milrinone's positive inotropic effect may induce rather than prevent ventricular arrhythmias in some patients.

A 24-year-old man comes to the office for hearing loss, tinnitus, and unsteadiness with walking. A CT is shown: https://i.imgur.com/Vj5PEDT.png. The patient is most likely to have a mutation in which gene? A. RB1 gene on chromosome 31 B. NF2 gene on chromosome 22

B. NF2 gene on chromosome 22 Bilateral vestibular schwannomas are the hallmark of neurofibromatosis type 2, a dominantly inherited disorder caused by mutations of the NF2 gene on chromosome 22. RB1 tumor suppressor gene mutations increase the risk of retinoblastoma and osteosarcoma.

An ultrasound probe is placed in the midesophagus and faced anteriorly. Which chamber is closest to the probe? A. Left atrium B. Right ventricle

A. Left atrium The left atrium form the majority of the posterior surface of the heart and resices adjacent to the esophagus. Enlargement of the left atrium can compress the esophagus and cause dysphagia. The right ventricle forms the anterior (sternal) surface of the heart and the majory of its inferior border on frontal chest x-ray.

A 48-year-old woman has painful, heavy menses for the past 6 months. She has had 4-5 days of heavy bleeding, often soaking through a tampon every few hours. Menstrual periods are regular every 28-30 days. Urine pregnancy test is negative. Hysterectomy is performed and shown here: https://i.imgur.com/7FgTXo2.png. What is the most likely diagonsis? A. Adenomyosis B. Uterine leiomyoma

A. Adenomyosis Adenomyosis typically presents with regular (cyclic) heavy, painful menses. Gross pathology reveals a uniformly enlarged, globular uterus with the abnormal presence of endometrial glands and stroma in the myometrium. Uterine leiomyomas (https://i.imgur.com/lBMqMGx.png) are benign, uterine smooth muscle cell tumors that can cause regular, heavy menses as well. However, gross patholkogy typically reveals an irregularly enlarged uterus with discrete tumors rather than a uniformly globular (boggy) uterus

A 48-year-old woman comes in for a follow-up of primary hypothyroidism due to chronic autoimmune (Hashimoto) thyroiditis. She feels well and takes levothroxine daily on an empty stomach. The thyroid is normal to palpation. Labs show serum TSH <0.01 uU/mL. The current dose of levothyroxine would put this patient at greatest risk for A. Atrial fibrillation B. Goiter

A. Atrial fibrillation Thyrotoxicosis, whether due to endogenous hyperthyrodism or over-replacement with thyroid hormone in patients with hypothyroidism, causes a hyperadrenergic state that can lead to aFib, high-output heart failure, and worsening of angina. Goiter is due to chronic thyroid stimulation by TSH, which improves with levothyroxine administration as TSH levels are decreased and is not seen in over-replacement.

A 60-year-old man has difficulty tasting food 2 months after a TBI after a MVC. He was in a coma for several days and began having severe headaches and impaired taste upon waking up. The headaches have improved, but he continues to have difficulty tasting food. No visual concerns, slurred speech, trouble swallowing, vertigo, or extremity weakness or numbness is reported. No focal motor deficits are noted on phsical. What is the most likely cause of these symptoms? A. Avulsion of olfactory nerve rootlets B. Lingual nerve injury

A. Avulsion of olfactory nerve rootlets Olfactory signals are relayed via the olfactory nerve (CN I) through the cribiform plate to the olfactory bulb, which then projects to the primary olfactory cortex in the medial temporal lobe. Head trauma can tear olfactor nerve rootlets as they cross the cribiform plate, causing anosmia. Anosmia is often interpreted by patients as loss of taste. Lingual nerve is a branch of the mandibular division of the trigeminal nerve (CN V3). It provides somatic sensation to the anterior 2/3s of the tongue. Fibers of the chorda tympani nerve, a branch of the facial nerve (CN VII), also travel with the lingual nerve to relay taste from the anterior 2/3s of the tongue. If taste disturbances were due to injury of the lingual nerve, tongue numbess would also be present.

A 64-year-old woman is in clinic due to a fall. She has had multiple ground-level falls in the last 6 months without significant injury. She gave up driving following a near-miss MVA a year ago. Past hx is notable for Type 2 DM and HTN, which is appropriately controlled. Opthalmic exam reveals reduced vision in her peripheral visual fields bilaterally and an elevated intraocular pressure. Timolol drops are initiated. Which structure is the target of this medication? A. Ciliary epithelium B. Trabecular meshwork

A. Ciliary epithelium Open-angle glaucoma is characterized by progressive loss of peripheral vision from elevated intraocular pressure. Timolol and other nonselective beta blockers work by diminishing the secretion of aqueous humor by the ciliary epithelium. Acetazolamide, a carbonic anhydrase inhibitor, also decreases aqeuous humor secretion by the ciliary epithelium. Prostaglanding F2a (latanoprost, travoprost) and cholinomimetics (pilocarpine, carbachol) decrease intraocular pressure by increasing the outflow of aqueous humor. Cholinergic agonists like pilocarpine cause miosis by promoting contraction of the sphincter of the iris, causing the anterior chamber angle to become wider and makes the trabecular meshwork more accessible to outflow of aqeuous humor.

A 40-year-old woman is brought in by her roommate with significant left leg weakness after her father had a heart attack, but everything is normal on exam and there's no identifiable causal factor. What is the most likely diagnosis? A. Conversion disorder B. Somatic symptom disorder

A. Conversion disorder Conversion disorder is characterized by neurologic symptoms and exam findings that are incompatible with a known neurological disease. Patients do not consciously produce the symptoms like in factitiious disorder or malingering, and the condition may cause significant distress/impairment. Somatic symptom disorder is characterized by excessive and persistent health anxiety and preoccupation with multiple somatic symptoms, it does not present with a clearly incompatible neuro exam. The patient in question does not express any concern about her impairment and seeks care only when brought in by her roommate, effectively ruling out somatic symptom disorder.

CD19 positive cells are isolated, purified, and exposued to monoclonal antibodies against cell surface complement receptor CD21. These cells are subsequently incubated along with several viruses. Initial exposure to monocloan antibodies against CD21 is most likely to prevent cell infection with which of the following viruses? A. EBV B. CMV

A. EBV The initial attachment of the virion envelope or capsid surface proteins to the complementary host cell surface receptors is essential to viral tropism for specific tissues and invasion of cells. Many viruses bind to normal host cell plasma membrane receptors to enter host cells. Known host cell receptor and virion/virion protein binding specificities include: CD4 with HIV gp120, CD21 with EBV gp350, and erythrocyte P antigen with parvovirus B19. CMV requires initial contact with glycosaminoglycan chains on host cell surface proteoglycans for entry into host cells, just like other herpesviruses. It does not require contact with CD21.

A patient becomes unresponsive and develops cardiac arrest. He is intubated, and sent to the ICU. Before discharge, it is noted that is dead space ventilation has increased. What contributed to this shift? A. Elevated RR, decreased TV B. Decreased RR, elevated TV

A. Elevated RR, decreased TV Patients being weaned from mechanical ventilations typically breathe at low tidal volumes, with a compensatory increase in RR to maintain minute ventilation. Because at low tidal volumes, a higher proportion of each breath is composed of dead space, this type of breathing typically leads to an increase in wasted ventilation (inefficient breathing). A decrease in RR and increase in TV would keep minute ventilation constant, but the total volume of dead-space air breathed each minute would decrease rather than increase

A 65-year-old woman has nausea, vomiting, and abdominal pain in the ER that began 6 hours ago. She has had vague pelvic pain over the last few months, but this episode is much more severe. Temp is 101F, BP is 141/90. Physical exam shows moderately distended abdomen. There is a tender bulge below the inguinal ligament, lateral to the pubic tubercle. The overlying skin is erythematous. What structure is immediately lateral to the bulge? A. Femoral vein B. Pectineal ligament

A. Femoral vein Femoral hernias can present with groin discomfort and a tender bulge on hte upper thigh inferior to the inguinal ligament, lateral to the pubic tubercle and lacunar ligament. The structure that lies immediately lateral to the hernia within the femoral sheath is the femoral vein. Incarceration and strangulation are common complications of femoral hernias. The pectineal ligament is a thickened part of pectineal fascia. It overlies the pectineal ridge of the pubic bne and is located posterior to the femoral canal.

A 37-year-old man due to blood-tinged vomiting and abdominal discomfort. He has been drinking large amounts of alcohol over the past 6 months on a daily basis. He has been hospitalized multiple times for alcohol intoxication. His vitals are normal, but physical exam shows a firm, enlarged liver. Peripheral blood smear shows neutrophils with 6-8 nuclear lobes. What is the most likely explanation for this smear? A. Folate deficiency B. Myelodysplasia

A. Folate deficiency Folate deficiency anemia commonly occurs in alcoholism. It is a megaloblastic anemia that can develop within weeks. Peripheral blood smear shows macrocytosis, ovalocytosis, and neutrophils with hypersegmented nuclei. Myelodysplasia manifests as pancytopenia, impaired blood cell differentiation, and clonal expansion of mutated hematopoietic cells in the bone marrow. Elevated MCV can be present, but hypersegmented neutrophils are not commonly seen

A 43-year-old man has a several-month history of fatigue, rash, flushing, and abdominal cramps. His rash is worse with rubbing or scratching, and he has diffuse itching after hot showers. He is frequently dizzy and light-headed after prolonged standing and had an episode of syncope working in the hot sun. Skin exam shows maculopapular rash, and biopsy shows large clusters of mast cells that are positive for KIT (CD 117). What else will be found? A. Gastric hypersecretion B. Pancreatic endocrine tumor

A. Gastric hypersecretion Systemic mastocytosis is characterized by abnormal proliferation of mast cells and increased histamin release. Histamine causes hypersecretion of gastric acid by parietal cells in the stomach as well as a variety of other symptoms (hypotension, flushing, pruritis). Pancreatic endocrine tumors may secrete gastrin (Zollinger-Ellison syndrome), insulin (hypoglycemia), glucagon (hyperglycemia, rash), somatostatin (diarrhea, cholelithiasis, hyperglycemia), or vasoactive intestinal peptide (watery diarrhea, hypokalemia, achlorhydria). These disorders would not be associated with excess mast cells in the skin.

A 43-year-old man has an early fall due to a 4-week history of cough. He initially had a "Bad cold" that lasted 10 days. The sneezing, runny nose, and muscle aches that accompanied the cold have improved, but the cough has persisted and worsened. He has "bursts of coughing" and feels he is unable to clear mucous. These coughing fits sometimes lead him to vomit. He received all childhood vaccinations but has not seen a physican in years. No history of recent travel. Chest X-ray is unrevealing. What is the most likely cause? A. Gram-negative coccobacillus B. Gram-positive diplococcus

A. Gram-negative coccobacillus Pertussis should be considered in any adult who has not had updated vaccination boosters. The clinical presentation is a paroxysmal cough lasting >2 weeks that is associated with post-tussive emesis or inspiratory whoop after a sever coughing episode. Strep pneumoniae, a gram-postive diplococcus, is the most common cause of pneumonia and typically presents with lobar consolidation on chest x-ray. The patient's negative chest x-ray and lack of fever make pneumococcal pneumonia unlikely.

A 31-year-old man has oral lesions that cause occassional discomfort during eating. He also reports fatigue and unintentional weight loss. The patient has no prior medical problems. Oral exam: https://i.imgur.com/f2WDdWJ.png. Lesions are easily scraped off revealing erythematous mucosa underneath. There are several enlarged cerival and axillary lymph nodes. What is the best diagnostic test for further workup for this patient? A. HIV antigen and antibody testing B. scraping microscopy for multinucleated giant cells

A. HIV antigen and antibody testing Oral thrush is caused by Candida albicans infection and usually presents as white, plaque-like oropharyngeal lesions that are easily scraped off with a tongue depressor. Affected patients typically have disruption to local microbial flora (eg antibiotic use) or impairment in cell-mediated immunity (corticosteroids, HIV). Young, otherwise healthy patients with thrush should be tested for HIV.

A 3-month-old girl has fever, irritability, and vomiting for 2 days. She is ill-looking, lethargic, and febrile. Blood cultures grow M. tuberculosis. One of her brothers died from disseminated mycobacterial infection during infancy. Impairment of which of the following mechanisms is contributing to the patient's infection? A. Interferon signaling B. Leukocyte adhesion

A. Interferon signaling Inherited defects involving INF-y signaling pathways result in disseminated mycobacterial disease in infancy or early childhood. Patients require lifelong treatment with antimycobacterial agents. LAD is a defect in CD18 (integrin) present in patients with delayed separation of the umbilical cord, poor wound healing, and recurrent cutaneous infections without pus formation.

A 58-year-old man has a large ulcerative mass in the middle third of his rectum extending to the rectosigmoid junction. He undergoes surgical resection of the rectosigmoid colon, and frozen section analysis reveals clear margins of the colon specimen with metastases in the pararectal lymph nodes. Which lymph node group is metastasis most likely? A. Internal iliac B. Left colic

A. Internal iliac Lymphatic drainage of the rectum proximal to the anal dentate line occurs via the inferior mesenteric and internal iliac lymph nodes. Areas distal to the dentate line drain primarily into the inguinal nodes. Left colic lymph nodes and their branches are located along the left colic artery and drain the left colonic flexure and upper descending colon.

A 67-year-old man has a persistent headache and pain in the jaw when chewing food. Hx reveals HTN, Type 2 DM, and hyperlipidemia. BP is 130/70. Physical exam is unremarkable. Appropriate therapy is started and an arterial biopsy is performed. Histopath shows multinuclear giant cells and internal elastic membrane fragmentation. Prompr institution of therapy is most likely to reduce th risk of which of the following? A. Ischemic optic neuropathy B. Retinal detachment

A. Ischemic optic neuropathy Giant cell arteritis (GCA) is characterized by granulomatous inflammation of the media with fragmentation of the internal elastic lamina most often affecting the medium and small branches of the carotid artery. Ischemic optic neuropathy with irreversible blindness is a potential complication of GCA; therefore, patients with suspected GCA require immediate glucocorticoid therapy. Retinal detachment presents with floaters, flashing lights called photopsia, and progressive monocular vision loss. Risk factors for retinal detachment include prior eye trauma or surgery, nearsightedness, age-related macular degeneration, and diabetic retinopathy.

A 25-year-old woman has a week of cough and dyspnea. She reports intermitten crampy abdominal pain and diarrhea. No history of respiratory illness or chronic disease, but recently was diagnosed with immune thrombocytopenia that resolved with high-dose glucocorticoid therapy. She is an immigrant from Myanmar 8 years ago. Temp is 99. Phys exam shows scattered wheezing and nontender abdomen. Lab studies reveal eosinophilia and sputum microscopy shows the following image: https://i.imgur.com/45dWeXe.png. What is the mode of transmission fo this pathogen? A. Larval skin penetration B. Parasite oocyst ingestion

A. Larval skin penetration Strongyloides stercoralis is transmitted in tropical/subtropical climates when human skin comes into direct contast with filariform larvae. The larvae penetrate the skin, enter the bloodstream, pass through the lungs, and are swallowed into the digestive system. Common manifestations include linear skin rash and irritative digestive/respiratory symptoms. Ingestion of oocytes, on the other hand, is a common mode of transmission for protozoan parasites like Giardia and Cryptosporidium. These guys often cause diarrhea and crampy abdominal pains too, but a nematode on sputum microscopy indicates a helminth (parasitic worm) infection. Helminths (strongyloidiasis, ascariasis, schistosomiasis) produce eggs containing multicellular embryos, not oocysts (Thick-walled spores)

A 3-year-old girl has worsening fever and lethargy. She's had recurrent upper respiratory illness with irritability and pulling at her ears. She has been eating poorly and vomiting and has become more lethargic. She had generalized tonic-clonic seizures in the ED. CT scan of her head shows a solitary 2-cm, ring-enhancing lesion in the left lateral temporal lobe. The pathogen responsible probably gained access to the brain from which of the following structures? A. Mastoid air cells B. Ethmoid air cells

A. Mastoid air cells A single brain abscess is usually caused by direct spread of a contiguous infection. Temporal lobe abscess is usually a result of otitis media spread to the mastoid air cells, while frontal lobe abscess is usually due to ethmoid or frontal sinusitis. Multiple brain abscesses typically indicate hematogenous dissemination of a distant infection (endocarditis, empyema)

15-y/o boy presents with worsening rash and profuse sweating for 1 week. Symptoms began gradually, first noticing sweating on a cold day. He has had difficulty sleeping and sensitivity to light. Exam shows anxious-appearing boy with hyperhydrosis and a significant erythematous maculopapular rash over the torso, thighs, and upper arms, along with pink macules over the palms and soles with associated dequamation. Neuro exam is positive for a slight tremor. Excessive exposure to which agent is responsible? A. Mercury B. Inhalants

A. Mercury Mercery impairs the breakdown of catecholamines, and toxicity can lead to excessive stimulation of the SNS. A desquamating hypersensitivity rash can also occur, and deposition in the CNS can result in personality changes, insomnia, and tremor. Inhalants (hydrocarbons) can produce CNS symptoms with agitation, but are more acute, severe (ataxia, disorienation, hallucinations). The inhalant rash is typically perioral,

A 68-year-old man has fatigue, weight loss, and constipation. He has not undergone any colonoscopies. Past medical hx is notable for emphysema and stable angina. He smoked a pack of cigs daily for 50 years and drinks alcohol occassionally. Colonoscopy reveals a circumferential mass encircling the sigmoid colon. Patho is positive for adenocarcinoma. CT scan of the body is negative for metastasis. Serum carcinoembryonic antigen assay is ordered, and its result would be most useful for which of the following aspects of this patient's care? A. Monitoring for residual disease B. Staging of the tumor

A. Monitoring for residual disease Carcinoembryonic antigen (CEA) levels are increased in colon cancer but are also elevated in a number of other conditions (pancreatic cancer, COPD, cirrhosis). CEA cannot be used to diagnose colon cancer, but it is helpful for detecting residual disease and recurrence. Elevated CEA are associated with a worse prognosis regardless of tumor stage, but CEA measurement is not currently considered part of the initial staging evaluation.

24-year-old man has swelling in his left legafter his dog bit him. The dog is fully vaccinated. The bite broke the skin and is bleeding. The area of the wound is warm, red, and swollen. Temp is 97.5F. Exam shows an open wound on the left leg with surrounding erythema, warmth, and tenderness. No wound drainage or crepitation is present. Distal pulses are palpable. Wound cultures grow gram-negative coccobacilli with a mouse-like odor. What organism is responsible? A. Pasteurella multocida B. Francisella tularensis

A. Pasteurella multocida Soft tissue infections following cat and dog bites are typically due to animal oral flora, the most common of which is Pasteuralla. Francisella tularensis is a gram-negative bacterium that causes tularemia, a zoonotic infection that presents with regional lymphadenopathy and an ulcerative lesion at the inoculation site. This infection typically occurs following contact with animals such as rabbits and rodents, not dogs.

An experimental animal is made with resected pituitary glands, resulting in decreased production of epinephrine from the adrenal medulla and cortisol frmo the adrenal cortex. Decreased activity of what enzyme is responsible for the lower epi? A. Phenylethanolamine-N-methyltransferase B. Monoamine oxidase

A. Phenylethanolamine-N-methyltransferase Cortisol increases the conversion of NE to EPI in the adrenal medulla by increasing the expression of phenylethanolamine-N-methyltransferase. MOA inactivates catecholamines (metanephrine and normetanephrine to vanillylmandelic acid).

A 25-year-old woman has an early anatomy ultrasound at 18 weeks gestation after she was noted to have elevated alpha fetoprotein. US shows a complex cystic lesion overlying the lumbosacral spine, ventriculomegaly, and hindbrain herniation. These abnormalities are due to failure of which process? A. Primary neurulation B. Neural crest cell migration

A. Primary neurulation Defects in primary neurulation result in open neural tube defects, such as myelomeningocele, in which the spinal cord and meninges protrude through a vertebral arch defect. Findings typically incldue an elevated maternal alpha fetoprotein and a complex cystic mass overlying the lower spine. Neural crest cells are the progenitors for the peripheral (NOT CENTRAL) nervous system; Hirschsprung disease (congenital aganglionic megacolon) is a disorder of neural crest cell migration. NTDs are not related to neural crest cell abnormalities.

39-year-old paraplegic man with an indwelling catheter has 24 hours of rigors, nausea, and vomiting. He is febrile and has suprapubic CVA tenderness. Urinalysis shows 3+ leukocyte esterase and numerous white blood cells. Urine and blood cultures grow non-lactose-fermenting Gram-negative rods. Which pathogen is the most likely culprit? A. Pseudomonas aeruginosa B. Escheria coli

A. Pseudomonas aeruginosa P. aeruginosa is an oxidase-positive, non-lactose fermenting, gram-negative organism. It is a common cause of UTI in patients with indwelling bladder catheters

A patient's anaphylactic reaction is mediated by antigen-specific IgE antibodies attached to high-affinity receptors on the surface of mast cells and basophils. What is the mechanism triggering vasoactive substance release? A. Receptor aggregation B. Antibody-receptor covalent binding

A. Receptor aggregation The high-affinity IgE receptor (FcERI) is found on the surface of mast cells and basophils and normally binds the Fc portion of circulating IgE antibodies. Cross-linking of multiple membrane-bound IgE antibodies by a multivalent antigen results in aggregation of the FcERI receptors, causing degranulation and the release of preformed mediators (histamine, tryptase) that initiate an allergic response.

A newborn has cyanosis 3 hours after delivery. Physical exam reveals subcostal retractions and grunting and different oxygen saturations in the right and left upper extremities. Cardiac exam reveals a 2/6 systolic murmur at the LLSB. Femoral pulses are strong bilaterally. What is the cause? A. Right-to-left shunt via PDA B. Right-to-left shunt via VSD

A. Right-to-left shunt via PDA Differential cyanosis (postductal < preductal oxygen saturation) in a neonaute suggests right-to-left shunting across a PDA, such as in persistent pulmonary hypertension of the newborn (PPHN). Patients with PPHN typically have an underlying pulmonary disorder with respiratory distress and strong femoral pulses. Postductal sat is arterial oxygen content while preductal is arterial oxygen in blood after it leaves the heart but before it reaches the ductus arteriosus of the aorta where it is sent to the upper extremities and the brain. In VSD, the preductal and postductal saturations would be equally low.

An experimental cisplatin drug encapsulated in liposomes coated with anti-EGFR antibodies. Patients given this uexperimental drug have a statistically significant decrease in residual tumor burden. New formulation does this by... A. Selective drug uptake by cancer cells B. Decreased drug efflux from tumor cells

A. Selective drug uptake by cancer cells Antibody-drug conjugates improve drug efficacy and minimize toxicity by allowing conventional chemo agents (cisplatin) to selectively target and kill cancer cells while sparing healthy cells (targeted delivery). The one in question will bind selectively to tumor cells expressing high levels of EGFR. Overexpression of drug efflux pumps (P-glycoprotein) by cancer cells is a major cause of chemo resistance. Efflux pump inhibitors can decrease drug excretion from tumor cells and potentiate effects of chemo agents

A 67-year-old man is found dead in his home. He had a long history of HTN and he had an MI a year ago. Gross exam of the heart shows white scarring and enlargement of the left ventricle. The type of collagen seen in the autoopsy is most likely to be found extensively in which normal tissue? A. Tendon B. Basement membrane

A. Tendon Type I collagen is the primary collagen in mature scars (ie, remote MI). It is the most prevalent type of collagen and provides strength and support throughout the human body, particularly in bones, tendons, ligaments, and skin. Basement membrane has Type IV collagen, as does hyaline collagen. Type II collagen is found in the nucleus pulposus.

34 year-old woman comes in after her sister was diagnosed with breast cancer. She has no breast abnormalities. Her sister was found to have a multiple base pair insertion affecting exon 11 of the BRCA1 gene leading to a frameshift mutation. A PCR test was performed. What must be known in order to perform amplification of this assay? A. The nucleotide sequence of the regions flanking the target exon B. Restriction enzyme susceptibility sites within the target exon

A. The nucleotide sequence of the regions flanking the target exon PCR is used to amplify small fragments of DNA by repeated replication. It requires primers that are complementary to the regions of DNA flanking the segment of interest. Thermostable DNA polymerase, deoxynucleotide triphosphates, and a source of DNA tempalte strand are also necessary. Restriction sites can be used to identify polymorphisms within variant alleles on gel electrophoresis (restriction fragment length polymorphism analysis). Not necessary for PCR tho.

A 1-week-old boy is born to a mom diagnosed and treated with gestational diabetes at 28 weeks. The nursery course was uncomplicated. His weight, length, and head circumference are 50th percentile. Phys exam reveals II/VI harsh, holosystolic murmur heard at the left med to lower sternal border. Birth records show no murmor was heard by 2 different health care providers in the newborn nursery. What is the most likely diagnosis? A. Ventricular septal defect B. Patent ductus arteriosus

A. Ventricular septal defect VSD typicallly presents in the neonatal period after pulmonary vascular resistance has declined. The clinical presentation depends on the size of the defect, which ranges from asymptomatic holosystoic murmur (small VSD) to heart failure (large VSD). The murmur is usually not detectable at birth, but becomes audible around 4-10 days later, permitting significant left-to-right shunting. Most small VSDs close spontaneously. PDA is a continuous murmur.

15-year-old girl with amenorrhea is found to have fully developed secondary sexual characteristics with a shortened vaginal canal and rudimentary uterus. what is the most likely diagnosis? A. Kallman syndrome B. Mullerian agenesis

B. Mullerian agenesis Mullerian agenesis presents with variable uterine development and no upper vagina (shortened vagina). These patients are 46,XX females with normal ovaries and secondary sexual characteristics. Kallman syndrome occurs due to decreased GnRH snthesis in the hypothalamus. Although female patients are rarely affected, they can have primary amenorrhea but typically do not develop secondary sexual characteristcs.

A 38-year-old man has generalized tonic-clonic seizure. He had several days of progressive headache, vomiting, and confusion prior to the episode. Temp is 100.4F. Physical exam shows lethargy with bilateral papilledema and neck rigidity. His condition worsens and the patient dies. Autopsy reveals this: https://i.imgur.com/OMXqAnH.png A. Yeast with polysaccharide capsule B. Bacteria with mycolic acid in cell wall

A. Yeast with polysaccharide capsule Cryptococcus neoformans is a round/oval yeast with a thick, washed-out polysaccharide capsule. It is a common cause of meningoencephalitis (headache, vomiting, confusion, seizure) in patients with untreated AIDS. The yeast can frequently be visualized in the CSF by India Ink or silver stain. M. tuberculosis (my answer) CAN spread to the brain and cause subacute meningitis or a focal parenchymal infection called a tuberculoma. But histo would show granulomas with giant cells and acid-fast bacteria: https://i.imgur.com/GnN4Ped.png

A 12-year-old boy was admitted 3 years ago after a non-fatal drowning event resulting in severe ARDS. He was intubated and mechanically ventilated for 1 week and discharged 2 weeks later. He now feels well and reports mild dyspnea with exertion. Which of the following is most likely increased in this patient compared to a child with normal cardiopulmonary functioning? A. alveolar-arterial oxygen difference B. diffusion capacity of the lungs

A. alveolar-arterial oxygen difference Survivors of ARDS commonly have reduced lung function that persists for months or years and is sometimes permanent. The majory of patients have reduced diffusion capacity that eventually normalizes after several years, whereas a small percentage develop extensive fibrosis with a marked and permanent REDUCTION in diffusion capacity.

A 32-year-old woman has pain and a palpable mass in the right flank. Imaging reveals a large mass arising from the right kidney and similar smaller masses in the left kidney. Surgey is performed and a cross section of the specimen is shown: https://i.imgur.com/hVzRiBT.png. Histo eval shows that the mass is composed of fat, smooth muscle, and blood vessels. This patient will most likely have which additional findings? A. brain hemartomas and ash-leaf skin patches B. cerebellar hemangioblastomas and liver cysts

A. brain hemartomas and ash-leaf skin patches Renal angiomyolipoma is a benign tumor composed of blood vessels, smooth muscle, and fat. Bilateral renal angiomyolipomas are associated with tuberous sclerosis, an autosomal dominant condition marked by brain hemartomas and ash-leaf skin patches. Cerebellar hemangioblastomas and liver cysts are associated with von Hippel-Lindau disease, an autosomal dominant disease with patients at high risk for bilateral renal cell carcinoma, which may grossly resemble renal angiomyolipoma, but microscopically shows tumor cells with abundnat clear cytoplasm.

A 53-year-old woman has progressive distention and tightness in her right lower extremity. She has had no shortness of breath, fever, or skin rash. She was diagnosed with melanoma of the right thigh 2 years ago and underwent surgical resection. Two of her inguinal lymph nodes tested positive for metastasis, and she subsequently underwent inguinal lymphadenopathy and received adjuvant therapy. Recent imaging reveals no evidence of recurrent malignancy. Hx includes hypertension and hyperlipidemia. Vitals are normal with no JVD. The right lower extremity is significantly larger in diameter than the left. Edema is present up to the thigh with mild skin indentation after application of pressure. The skin overlying the involved area is thickened and dry. Lab testing, including CBC, metabolic panel, and urinalysis are normal. Treatment of this patient's symptoms should include which of the following? A. compression to increase lymphatic flow B. reduction of intravascular volume

A. compression to increase lymphatic flow Chronic lymphedema is most commonly caused by an acquired disruption of lymphatic drainage (eg due to malignancy or lymphadenectomy), and typically presents with swelling and thickened skin in one or more extremities. Treatment is usually conservative and involves compression bandages and physiotherapy; diuretics are ineffective and contraindicated.

Novel drug X is primarily eliminated by the kidneys. A healthy volunteer weighing 60 kg has a GFR of 100 mL/min. Immediately after IV administration plasma concentration of drug X is 0.5 mg/mL. Urinary excretion rate of the drug is found to be 75 mg/min. When drug Y is coadministered, urinary excretion rate of drug X is 50 mg/min. Which of the following effects of drug Y on pharmacokinetics of drug X best explains these findings? A. decreased renal tubular secretion B. displacement from plasma proteins

A. decreased renal tubular secretion renal excretion of a drug is dependent on glomerular filtration, renal tubular secretion, and tubular reabsoprtion. Glomerular filtration is reduced with low RBF, kidney disease, and high drug protein binding. Renal tubular secretion is reduced by coadministration of drugs with overlapping substrate specifity. Tubular reabsorption may be altered by changes in urine pH. Displacement from plasma proteins would increase the excretion rate of drug X.

A 26-year-old woman has worsening generalized weakness, myalgias, and unintentional weight loss. She has primary hypothyroidism, which she takes levothryroxine for. Her BP is 110/70 sitting and 90/60 standing. She is mildly emaciated. Lab studies reveal mild normochromic, normocytic anemia, eosinophil count of 15%, serum glucose of 65 mg/dL. What else will be found in the patient? A. decreased serum Na, increased serum K, increased urine Na, decreased urine K B. decreased serum Na, increased serum K, decreased urine Na, increased urine K

A. decreased serum Na, increased serum K, increased urine Na, decreased urine K This is primary adrenal insufficiency (Addison disease), which can occur in autoimmune diseases like hypothyroidism. Reduced aldosterone --> renal salt wasting --> hypovolemia and orthostasis. Reduced cortisol --> hypoglycemia, normocytic anemia, and eosinophilia. Reduced aldosterone production in primary adrenal insufficiency leads to renal sodium wasting with consequent hypovolemia, orthostasis, and potassium retention (hyperkalemia). Low cortisol stimulates increased ADH secretion, which leads to water retention and hyponatremia.

A 66-year-old man comes to the office for hypertension followup. He takes lisinopril and amlodipine, but his BP has been 140-150/85-95 for the past 2 visits. He also reports worsening urinary symptoms over the past year that include hesitancy, straining during urination, poor urinary flow, and waking up once or twice a night to urinate. He feels the symptoms are bothersome but are not affecting his quality of life. His BP is 142/88 today. Physical exam reveals mildly enlarged, smooth prostate. What is the most effective medication for both medical issues? A. doxazosin B. finasteride

A. doxazosin Alpha-1 blockers such as doxazosin, prazosin, and terazosin are useful for the treatment of both benign prostatic hyperplasia and hypertension. To minimize adverse effects and drug interactions, it is desirable to prescribe a medication that can address multiple issues at once. Finasteride is a 5-alpha-reductase inhbitor that is effective for the treatment of BPH but has no effect on BP. It will decrease the size of the prostate though.

An 11-year-old girl is evaluated for blurry vision. She was adopted internationally at age 9. She is at the 40th percentile for height and weight. Other than bilateral lens opacities, the rest of her exam is normal. Urine is positive for reducing substances. Deficient activity of which enzyme is the cause for the patient's eye condition? A. galactokinase B. hexosaminidase A

A. galactokinase Lenticular accumulation of galactitol in the lenses of patients with galactosemia can cause osmotic damage and development of cataracts. Cataracts are frequently the only manifestation of galactokinase deficiency. Hexosaminidase A deficiency is Tay-Sachs.

A 26-year-old has a 6-year history of generalized tonic-clonic seizures. A year ago, she had partial loss of vision due to an occipital infarction. Her mother has chronic intermittent muscle weakness and lactic acidosis, and her maternal uncle has hemiplegia. The patient's SKM biopsy shows muscle fibers with irregular contours when stained with Gomori trichrome. After further eval, all affected family members have the same inherited disease. Pedigree analysis is as shown: https://i.imgur.com/QXZgMyV.png. What is the most likely explanation for the variability of clinical manifestations in the affected family members? A. heteroplasmy B. locus heterogeneity

A. heteroplasmy Mitochondrial encephalomyopathy is characterized by ragged red fibers in muscle biopsy and a maternal inheritance pattern. Clinical presentation in affected family members varies due to the degree of heteroplasmy, or proportion of mitochondria with normal and mutant genomes in the same cell or tissue. Locus heterogeneity describes mutations that occur at different loci (ie different genotypes) but cause similar phenotypes. In contract, family members affected by a mitochondrial disease likely have the same genotype but a variable phenotype due to heteroplasmy

A 5-month-old boy has right-sided scrotal enlargement that has been present since birth that increases when he cries or strains. His parents do not believe in his pain, and they have noted discoloration of the area. There is no history of trauma or infection. On US, enlargement is found to be a fluid collection around the right testis. The specific embryologic defect giving rise to this patient's condition can also lead directly to which of the following? A. indirect inguinal hernia B. testicular torsion

A. indirect inguinal hernia Communication hydroceles and indirect inguinal hernias are caused by an incomplete obliteration of the processus vaginalis. The resultant connection between the scrotum and abdominal cavity can allow for fluid leakage (hydrocele) or the passage of abdominal contents (indirect inguinal hernia). Trauma or congenitally-horizaontal positioning of the testes (bell clapper deformity) facilitates testicular torsion.

A 33-year-old man has trouble swallowing and chronic sore throat. Exam shows enlarged, ulcerated right tonsil. Biopsy reveals infiltrating nests of moderately differentiated squamous cells. Immunochemistry is positive for p16, suggesting HPV as the etiology. Further imaging studies for cancer staging are planned. The tumor will most likely spread to where? A. jugular lymph nodes B. lung parenchyma

A. jugular lymph nodes Head and neck squamous cell carcinomas typically spread first to the anterior cervical (ie jugular) lymph nodes via the lymphatics. Distant spread (lung parenchyma) occurs after regional lymph node involvement.

A 56-year-old woman is brought to the ER with a 2-day history of high fever, headache, mild confusion, and dry cough. She has mild abdominal discomfort and watery diarrhea. The patient recently returned from a cruise to Hawaii. Her medical problems include hypertension and hyperlipidemia. She has a 20 pack/year smoke history. Her temp is 104F, BP is 104/63, pulse is 85, and RR are 24. Lung exam reveals lower lobe crackles with no wheezing. Abdomen is soft, non-destined, and non-tender. CXR reveals bilateral lower lobe interstital infiltrates. What additional finding is most likely? A. low serum sodium B. gram-positive diplococci on sputum Gram stain

A. low serum sodium Legionella pneumophila is a facultative intracellular gram-negative bacillus that can cause a systemic infection. Symptoms frequently include high fever, cough, confusion, and diarrhea. The most common lab abnormality seen with legionella pneumonia is hyponatremia, and sputum Gram stain often shows many neutrophils but few or no organisms. Strep pneumoniae is the most common overall cause of community-acquired pneumonia. Lobar consolidation is a classic finding. BUT dry cough, GI, and neurological symptoms w/ interstitial infiltrates in the setting of a recent cruise makes Legionella more likely.

A 34-year-old man has 2 weeks of progressive global headache, malaise, and fevers. He has had unintentional weight loss over the past several months. He has a history of injection drug use and was diagnosed with HIV 3 years ago. He has not been compliant with antiretroviral therapy. He is febrile and exam shows white patches on the oropharyngeal mucosa and mild generalized lymphadenopathy. Brain imaging reveals no intracranial masses. India ink staining of CSF reveals spherical yeast forms with thick capsules. What is the most likely site of primary infection in this patient? A. lungs B. skin

A. lungs Cryptococcus neoformans is a yeast with a thick polysaccharide capsule that is found primarily in soil contaminated by bird droppings. Inhalation of this pathogen into the lungs leads to asymptomatic primary infection usually followed by life-long latent infection. However, immunocompromised (advanced AIDS) patients are at high risk for reactivation with spread to the CNS. Skin bacteria are more commonly Staph aureus and strep pyogenes, they can cause systemic infection following skin breakdown. Systemic infection from skin fungus (Candida) following skin breakdown is uncommon. This patient's encapsulated yeast on India ink stain indicates Cryptococcus, making inhalation the most likely route of infection.

A 62-year-old woman has a change in bowel habits. For 2 months, she has had bowel movements once every 5-6 days with hard and lumpy small-volume stools. Defecation is uncomfortable. She has no trouble passing gas. She was admitted 3 months ago for acute diverticulitis, and her hospital course was complicated by new-onset atrial fibrillation. Colonoscopy a month after hospitalization revealed sigmoid diverticulitis but no masses or polyps. The patient takes fiber supplements, diltiazem, and apixaban. Cardiac auscultation reveals irregular heart rate but no murmur. Abdominal exam shows mild distention. Which of the following best explains this patient's symptoms? A. medication adverse effects B. gastrocolic reflex impairment

A. medication adverse effects Constipation is a common adverse effect of non-dihydropyridine calcium channel blockers (diltiazem, verapamil). Other medications that cause constipation include opiates, 5-HT3 antagonists, iron tablets, aluminum-containing antacids, and medications with anticholinergic properties. Gastrocolic reflex is a neurogenic reflex triggered by gastric distention of food. Impairment of this reflex CAN cause constipation, but this typically occurs with diabetic neuropathy and gastroparesis.

A 1-week old boy is evaluated for urosepsis and is receiving antibiotic therapy. The mother received no prenatal care, but the neonate was born at term. Vital signs are normal. Exam shows lower abdominal distention with normal bowel sounds. Penis and scrotum appear normal. Renal ultrasonograph and voiding cystourethrography reveal diffusely-thickened bladder wall with bilateral vesicoureteral reflux and hydronephrosis. What is the most likely cause of these findings? A. persistence of the urogenital membrane B. congenitally short intravesical ureters

A. persistence of the urogenital membrane Bladder outlet obstruction leads to increased pressure proximally and findings of bladder distention and wall thickening, secondary vesicoureteral reflux, ureteral dilation, and hydronephrosis. The most common cause in newborn boys is posterior urethral valves due to a persistent urogenital membrane. Primary vesicoureteral reflux is due to congenitally short intravesicular ureter that fails to fully compress during micturition, causing retrograde urine flow. Ureteral dilation, hydronephrosis, and UTIs may develop, but the bladder is distal to the defect and would not be distended or thickened.

A 54-year-old man comes to the office with daytime sleepiness and lack of energy. The symptoms began 6 months ago and have progessively worsened to the point of feeling "completely drained" by the end of the day. The patient's wife reports that he snores loudly. His past medical history is unremarkable but he has not seen a physician in over 10 years. He works in a warehouse of an agricultural supply company. BMI is 34. Physical exam shows a narrow oropharynx and a large neck circumference. He is at increased risk of developing which of the following? A. pulmonary hypertension B. narcolepsy

A. pulmonary hypertension Obstructive sleep apnea presents in obese individuals with excessive daytime sleepiness and signs of nocturnal upper airway obstruction (snorting, grasping). The condition is associated with systemic hypertension. Prolonged, untreated obstructive sleep apnea can also cause pulmonary hypertension and right heart failure. Like obstructive sleep apnea, narcolepsy can cause daytime drowsiness. However, narcolepsy is also associated with cataplexy (sudden loss of muscle tone), sleep attacks, sleep paralysis, and hypnagogic hallucinations. OSA is not a risk factor for narcolepsy.

A 72-year-old man has 3 hours of persistent retrosternal chest pain and dyspnea, and is diagnosed with acute MI. He undergoes cardiac catheterization, where the arterial wall of the right common femoral artery is penetrated superior to the right inguinal ligament. Soon after a corrective procedure, the patient becomes cold, clammy, and hypotensive. Physical exam shows a 5 cm region of ecchymosis surrounding the femoral puncture site. Internal bleeding is suspected. Which of the following is the most likely location of the blood collection? A. retroperitoneal space B. pelvic cavity

A. retroperitoneal space The optimal site for obtaining vascular access in the lower extremity during cardiac catheterization is the common femoral artery below the inguinal ligament. Cannulation above the inguinal ligament can significantly increase the risk of retroperitoneal hemorrhage. Bleeding into the pelvic cavity is frequently due to trauma (pelvic fracture) or gynecologic hemorrhage (ruptured ectopic pregnancy). Injury to the common femoral artery is less likely to cause intraperitoneal bleeding as the external iliac artery courses underneath the peritoneum.

A 42-year-old man has numbness and tingling in both legs and difficulty walking for a few months. He tires easily, has a temp of 98.2 F, BP is 122/86, pulse is 76, and RR are 14. Exam shows conjunctival pallor and loss of vibration and position sensation in the bilateral lower extremities with associated gait ataxia. What is likely to be present on further questioning of the patient? A. strict vegan diet for the past 6 years B. total gastrectomy 8 months ago

A. strict vegan diet for the past 6 years Vitamin B12 wis obtained through diet solely from animal sources, which places strict vegans at risk for dietary deficiency. This deficiency takes years to develop due to the large hepatic vitamin B12 reserve, and presents with megaloblastic anemia and potentially irreversible neurologic deficits (eg paresthesia, weakness, ataxic gait). The liver can store approximately 1000 times the daily vitamin B12 requirement; deficiency develops only after the complete absence of intake or absorption for 3-4 years, making a longstanding (6 years) vegan diet consistent with this patient's presentation, in contrast to a recent (8 months) gastrectomy.

A 21-year-old man has intense pruritis of the inguinal-genital area. He has no fever, dysuria, or skin rash. He has had 3 sexual partners and was treated for chlamydial urethritis 6 months ago. Since then he has been using condoms consistently. GU exam shows no urethral discharge, penile lesions, or inguinal lymphadenopathy. Skin excoriation marks in the pubic area and other findings are shown here: https://i.imgur.com/TNCD5ZT.png. Which of the following is the most appropriate pharmacotherapy for this patient's condition? A. topical permethrin B. oral doxycycline

A. topical permethrin Phthirus pubis is the human pubic louse. It is transmitted sexually via skin-to-skin contact and causes intense pruritis. Louse and nits (eggs) can often be visualized on exam. Treatment is with topical permethrin, which blocks parasite sodium ion conduction in nerve cell membrane channels, resulting in louse paralysis and death.

A 26-year-old woman has fever, malaise, and joint pain. A week ago, the patient had pain in the small joints of her right hand, which improved, but was followed by right ankle and left wrist pain. Temp is 100.4F. Extremity joints are not erythematous or swollen, but tenderness is present along the tendons of the left wrist and right ankle. Several small, nontender pustules are seen on the extremities. There is no other rash. Blood culture in selective media of chocolate agar and multiple antibiotics yields the causative pathogen. Which of the following strategies would have prevented this condition? A. using condoms consistently B. obtaining adequate vaccinations

A. using condoms consistently N. gonorrhoeae is spread sexually by GU secretions and can be prevented by consistent condom use. GU infections are often asymptomatic, which increases the risk of spread through the bloodstream, leading to disseminated gonococcal infection. Patients with disseminated disease typically present with the triad of polyarthralgia, tenosynovitis, and dermatitis or purulent arthritis. N. gonorrhoeae undergoes dramatic antigenic variation of its surface molecules, preventing the formation of protective antibodies, so vaccinations are ineffective.

A 14-year-old boy has severe, prolonged bleeding following a tooth extraction. He has a history of multiple episodes of painful joint swelling following minor trauma. His parents have no bleeding problems, but one is a carrier for one. His older sister does not have the condition, but is pregnant and does not know the sex of the child. What is the risk that her child will have the disease? A. 1/2 B. 1/8

B. 1/8 Hemophilia is an X-linked recessive disease. The probability that a female sibling of a male affected by an X-linked recessive disease will give birth to an affected child is 1/8. The probability that the sister i a carrier is 1/2. The probabiltiy that the offspring of the female carrier will inherit the X chromosome with the hemophilia gene is 1/2. The probability that the offspring is a boy is 1/2. The probability that all 3 events will take place is 1/8.

A new drug Superstatin is evaluated. In patients treated with Superstatin, 10 experience an MI while 990 do not. In patients treated with the control medication, 25 experience MI while 975 do not. What is the NNT for Superstatin? A. 23 B. 67

B. 67 NNT = 1/absolute risk reduction = 1/(Control event rate - Experimental event rate). The control event rate is 25/1000 = 0.025. The experimental event rate is 10/1000 = 0.01. So the NNT = 1/(0.025 - 0.01) = 1/0.015 = 66.7

A 52-year-old man has right shoulder pain. He is an avid golfer but hasn't played for 3 months due to pain. It's started to interfere with his daily activities. An MRI shows thickening and calcifications of the supraspinatus tendon. Which of th efollowing shoulder actions will provoke pain? A. Internal rotation B. Abduction

B. Abduction The most commonly injured structure in rotator cuff syndrome is the tendon of the supraspinatus muscle. Because the supraspinatus is an abductor of the humerus, injury to its tendon causes pain on abduction of the arm. Internal rotation is accomplished by the subscapularis muscle.

A 45-year old man has 3 months of exertional fatigue and weight loss. He is sexually active with multiple male partners. Temp is 100.2. The patient appears chronically ill and has mucosal pallor. There are thick, white plaques over the buccal mucosa and soft palate. Anterior and posterior cervical lymphadenopathy is present. Lungs are clear and no organomegaly is noted. Lab shows slightly microcytic MCV with low serum iron and low TIBC. What is the most likely cause of this patient's anemia? A. GI blood loss B. Abnormal iron utilization

B. Abnormal iron utilization Anemia of chronic disease is caused by chronic elevations in inflammatory cytokines, leading to reduced iron absorption and increased iron retention in the reticuloendothelial system. This impairs iron availability and causes a normocytic or slightly microcytic anemia with low serum iron and low TIBC. Iron deficiency anemia (GI bleeding) also presents with microcytic anemia and low serum iron, but is associated with high TIBC due to increased circulating ferritin levels.

A toxic insult targets kinesin. What will be absent from tissues on histological exam? A. Desmosomes in epithelial lining cells B. Secretory vesicles in nerve terminals

B. Secretory vesicles in nerve terminals Kinesin is a microtubule-associated, ATP-powered motor protein that facilitates the anterograde transport of neurotransmitter-containing secretory vesicles down axons to synaptic terminals. Desmosomes are protein aggregates that mediate cell-cell adhesion between epithelial cells.

A 46-year-old man has an uncomfortable heartbeat sensation at night that decreases when he sleeps on his side. He has mild shortness of breath with exertion for 6 months, but denies chest pain. He was told 2 years ago he has a heart murmur. Physical exam shows bounding femoral pulses and carotid pulsatations accompanied by head bobbing. What is the most likely diagnosis? A. Coarctation of the aorta B. Aortic regurgitation

B. Aortic regurgitation Aortic regurgitation causes an increase in total stroke volume with abrupt distention and rapid falloff of peripheral arterial pulses, resulting in a wide pulse pressure. This leads to bounding peripheral pulses and head bobbing with each heartbeat. Coarctation of the aorta causes systolic HTN in the upper extremities along with characteristic diminished and/or delayed femoral pulses (brachial-femoral delay)

A 24-year-old previously healthy woman has a 3-day history of fever, dyspnea, and cough productive of yellow sputum. Temp is 101.8F. The patient has bronchial breath sounds and crackles over the RLL. Lab results show high leukocytes (54,000) with 10% band forms, 3% myelocytes, 1% metamyelocytes, and 15% lymphocytes. LAP is elevated. What is the most likely finding on a peripheral blood smear? A. Numerous blasts with azurophilic rods B. Basophilic oval inclusions in mature neutrophils

B. Basophilic oval inclusions in mature neutrophils Leukemoid reaction is a significant leukocytosis that may exceed 50,000 that occurs in response to an underlying condition, commonly severe infection. Blood smear often shows neutrophils with reactive features (Dohle bodies) as well as increased neutrophil precursors (bands, metamyelocytes, myelocytes). The leukocyte alkaline phosphatase score is normal or increased in leukemoid reaction and decreased in leukemia. Auer rods are seen in AML, but the patients lab reports show mainly mature neutrphils with increased neutrophil precursors. BLASTS would indicate AML.

A 14-year-old girl is at the 5th percentile for height and 25th for weight. Physical exam shows a short and thick neck, a broad chest, and shortened fourth metacarpals bilaterally. A murmur is heard on cardiac exam. Which of the following would most likely be seen on echocardiogram? A. Mitral stenosis B. Bicuspid aortic valve

B. Bicuspid aortic valve Turner syndrome is associated with congenital anomalies of the aorta, and the most common defect is a bicuspid aortic valve. A nonstenotic bicuspid aortic valve can manifest as an early systolic, high-frequency click over the right second interspace. Bicuspid aortic valves are at risk for stenosis, insufficiency, and infection. Mitral stenosis is seen in rheumatic heart disease, producing an "opening snap" mid-diastolic, low-pitched rumbling murmur.

A 62-year-old man has an intensely pruritic facial rash for 3 days. He frequently dyes his hair, and after doing so 5 days ago he recalls developing a similar rash. He has a history of asthma, HTN, and DM. Physical exam reveals the following: https://i.imgur.com/yCU2LgY.png. Which of the following is the primarily pathogenesis of the patient's rash? A. Mast cells and histamine B. CD8+ T cells and interferon gamma

B. CD8+ T cells and interferon gamma Allergic contact dermatitis is a type IV (delayed-type) hypersensitivity reaction. Initially, Langerhans cells present haptens to naive T cells, leading to clonal expansion. On reexposure, sensitized CD8+ T cells are recruited to skin and destroy tissue. Release of INF-y by T cells further amplifies the immune response. Mast cells mediate the type I hypersensitivity reaction. On reexposure, allergens bind to allergen-specific IgEs on mast cells, causing immediate granulation and release of vasoactive peptides resulting in uticaria, and anaphlyaxis if severe enough. Onset is rapid (minutes) unlike this patient's delayed (48-hr) response. Furthermore, individual wheals of urticaria resolve within 24 hours, rather than days.

The greatest difference in oxygen consumption is between the aorta and... A. Pulmonary Artery B. Coronary sinus

B. Coronary sinus Myocardial oxygen extraction exceeds that of any other tissue, so the cardiac venous blood in the coronary sinus is the most deoxygenated blood in the body. Due to the high degree of oxygen extraction, increases in myocardial oxygen demand can only be met by an increase in coronary blood flow. Deoxygenated systemic blood returning via the vena cava contains more oxygen than coronary sinus venous blood. Because the pulmonary artery contains a mixture of blood returning via coronary sinus and systemic venous circulation, its oxygen content is also higher than pure coronary venous return

A 65-year old man has pain, redness, and swelling in his right calf. He was diagnosed with cellulitis and clindamycin is initiated. A few days later, he develops watery diarrhea and abdominal cramps. CBC reveals leukocytosis. The toxin responsible for this patient's condition most directly affects which of the following components of intestinal mucosal cells? A. Apical iron transport B. Cytoskeleton integrity

B. Cytoskeleton integrity C. difficile can proliferate due to loss of commensal gut flora following the use of broad-spectrum antibiotics like clindamycin. C dif toxins A and B stimulate an inflammatory reaction and disrupt the actin cytoskeletal structure, resulting in pseudomembranous colitis characterized by crampy abdominal pain, watery diarrhea, and leukocytosis. Apical ion transport is most directed affected by cholera toxin (V. cholerae) through activation of adenlyate cyclase leading to decreased salt reabsorption and increased transport of sodium and chloride out of the gut mucosal cell. Apical ion transporters are indirectly affected by C dif toxins due to loss of cell polarity (secondary to cytoskeletal deformity)

A 58-year-old man is seen 6 months after stent placement. His father died at 50 from CAD. He has a healthy diet and is compliant with his meds. BMI is 29. Exam is unremarkable. After extensive discussion with the patient, evolocumab is added to the regiment. What is the expected immediate result of this treatment? A. Decreased intestinal cholesterol absorption B. Decreased low-density lipoprotein lysosomal degradation

B. Decreased low-density lipoprotein lysosomal degradation Binding of PCSK9 to LDL-R on hepatocyte cell surface increases degradation of LDL-R, leading to decreased uptake of circulating LDL. PCSK9 inhibitors (evolocumab, alirocumab) decrease LDL-R degradation, resulting in greater uptake of LDL in the liver and lower circulating LDL levels. Ezetimibe is a lipid-lowering drug that directly inhibits absorption of cholesterol in the small intestine. PCSK9 inhibitors do not do this.

A 23-year-old is evaluated for prolonged paralysis following general anesthesia, which was induced with propofol, succinylcholine, and maintained via propofol and fentanyl infusions. The patient has not had a reutn of neuromuscular function after several hours. She has been "slow to wake up" from anesthesia. Why the prolonged paralysis? A. Decreased first-pass metabolism B. Decreased plasma hydrolysis

B. Decreased plasma hydrolysis Succinylcholine, a depolarizing NMJ blocking agent, is rapidly hydrolyzed by plasma pseudocholinesterase. Patients with pseudocholinesterase deficiency have prolonged NMJ paralysis after drug administration. Patients with pseudocholinesterase deficiency are unable to metabolize succinylcholine, leading to large amounts of the drug reaching the NMJ. Heterozygotes may experience double the normal period of paralysis, while homozygotes can have persistent paralysis for several hours. Decreased first-pass metabolism occurs when drugs are administered via a nonoral route. Although succinylcholine is given IV, it is primarily inactivated by the plasma by pseudocholinesterase, not the liver

A 42-year-old woman has fever and chills after hemodialysis. She has a history of end-stage kidney disease due to IgA nephropathy. Hx includes depression for whcih she takes citalopram. Temp is 101.4 and BP is 130/80. There is no erythema at the catheter site. Blood cultures are obtained, and she is treated with vancomycin and ceftazidime. While recieving the IV infusion, the patient reports a burning, itching sensation. Vital signs are unchanged, but a erythematous rash involving the face and neck is observed. She reports no history of drug allergy but has never received these antibiotics. What is the most likely underlying cause? A. Serotonergic drug interaction B. Direct mast cell activation

B. Direct mast cell activation Vancomycin infusion reaction (flushing, pruritis, erythematous rash) is the most common adverse reaction to vancomycin. It occurs due to rapid vancomycin infusion, which leads to the direct activation of mast cells (non-IgE mediated) and the subsequent release of vasoactive mediators. Serotonergic drug interactions are most common with drugs that affect serotonin system like SSRIs, TCAs, and antiemetics like ondansetron. However, most cases are marked by alterations in autonomic function (HTN, tachycardia, hyperthermia), and rash is unusual

A patient is admitted for exacerbation of COPD w/ hx of CAD, type 2 DM, and obesity. He smokes cigs daily for 50+ years. The admitting physician continues his existing meds for diabetes and heart disease. Admission orders also include supplemental oxygen, bronchodilators, antibiotics, and systemic corticosteroids. Which of the following additional components is most critical to include durinf the admissions process? A. Discussion regarding smoking cessation to reduce the patient's risk of dying from heart or lung disease B. Discussion of whether the patient has advance directives or a health care proxy

B. Discussion of whether the patient has advance directives or a health care proxy The admission process must include discussion of whether the patient has advance directives, along with code status, to ensure that medical personnel adhere to the patient's specific wishes should the patient become incapacitated.

A 23-year-old woman has 2 hours of fever, chills, headache, and myalgias. She was treated for a genital ulcer earlier that day after a positive VDLR and fluorescent treponemal antibody absorption test. The remainder of her testing was negative. She received IM benzathine penicillin G and several hours later her current symptoms began. Temp is 100.4F. Lungs are normal. There is an indurated, nontender vulvar ulcer and several enlarged inguinal lymph nodes. No other skin rash is present. What is the etiology of this patient's symptoms? A. Conjugation of antibiotic and host antigens B. Drug-induced bacterial cell wall disintegration

B. Drug-induced bacterial cell wall disintegration Jarisch-Herxheimer reaction is an acute inflammatory reaction that occurs within hours of treatment for spirochetal infections (syphilis). The rapid lysis of spirochetes releases inflammatory bacterial lipoproteins into the circulation and causes acute fever, rigors, and myalgias.

An 18-month-old, partially vaccinated boy is set to receive the H. influenzae serotype b (Hib) conjugate vaccine. The parents ask why a tetanus toxoid conjugate is present. Which of the following best describes the purpose of Hib vaccine conjugation? A. Induces immunity against the conjugated toxoid B. Elicits a T cell-dependent immune response

B. Elicits a T cell-dependent immune response The H. influenzae serotype b vaccine consists of a capsular polysaccharide conjugated to a carrier protein (tetanus toxoid protein or outer membrane protein of N. meningitidis). Protein conjugation causes a T-cell mediated immune response leading to long-term immunity through production of memory B-lymphocytes.

A 43-year-old man has a 3-day history of persistent headaches. He has a history of HTN. His BP is 224/115 and he appears mildly confused, but no focal neurological deficits are noted. Funduscopic exam shows bilateral papilledema. Serum creatinine is 1.4. An IV medication is administered that causes arteriolar dilation while also improving renal perfusion and increasing natriuresis. What is the most likely drug? A. Nitroglycerin B. Fenoldopam

B. Fenoldopam Fenoldopram is a selective peripheral dopamine-1 receptor agonist. It causes vasodilation fo systemic and renal arterioles to lower BP while also increasing renal perfusion, urine output, and natriuresis, making it useful in the treamtent of hypertensive emergency in patients with renal insufficiency. Nitroglycerin is a rapid-acting venodilator that decreases preload and CO. It causes only minimal arterial dilation and does not improve renal perfusion. It is primarily used to reduce myocardial oxygen demand in acute coronary syndrome.

A 34-year-old woman has recent onset of malaise. The patient reports feeling "run down and under the weather" but otherwise has no symptoms. She is a nurse at a local hospital and lives at home with her husband and 2-year-old son. She occassionally smokes and goes out, but since began feeling ill, she no longer has the desire to smoke. Physical exam is notable for hepatomegaly. Lab results indicate positive Anti-HAV IgM and Anti-HBs, and negative everything else. What is the most likely etiology elicited on further history taking? A. Had an accidental needlestick exposure at work B. Had steamed oysters at a neighborhood restaurant

B. Had steamed oysters at a neighborhood restaurant Transmission of HepA occurs though the fecal-oral route and is common in areas with overcrowding and poor sanitation. Outbreaks frequently occur from contaminated water or food, and raw or steamed shellfish is a common culprit in the US. Accidental needlestick would be a risk factor for HepB and HepC. The positive lab for HBsAb confirms she has antibodies to HepB surface antigen and, given the negative anti-HBc, suggests she has been vaccinated and is now immune.

A 45-year-old woman collapses on a hot summer day. She has a history of fibromyalgia and takes amitriptyline. Temp is 104.9. The skin is warm and red. Her BP is 90/60 mmHg. If this patient's medication contributed to her current condition, which of the following mechanisms is responsbile? A. Increased peripheral vasoconstriction B. Impaired dissipation of body heat

B. Impaired dissipation of body heat Nonexertional heat stroke (NHS) is a life-threatening disorder characterized by hyperthermia (typically >40 C, or 101F) associated with CNS dysfunction (encephalopathy, syncope). Anticholinergic medications promote heat stroke by impairing diaphoresis. Other medications associated with NHS include sympathomimetics, dopamine antagonists, diuretics, and beta blockers. Sympathomimetics impair peripheral vasodilation, limiting heat transfer to the skin.

A 53-year-old man has difficulty breathing and increased fatigue along with shortness of breath at night and has been sleeping in a recliner. Hx is remarkable for HTN and hyperlipidemia. He had an MI that was not revascularized 2 months ago. He has a 25 pack/year hx. A low-frequency diastolic heart sound is heard shortly after the second heart sound when the patient liest in the left lateral decubitis position. Cardiac imaging shows hypokinesis of the left ventricular free wall. Which of the following will accentuate this patient's abnormaly auscultation finding? A. Valsalva strain phase B. Listening at the end of expiration

B. Listening at the end of expiration An S3 is a low-frequency sound occuring during early diastole, just after S2. Left ventricular gallps (S3 and S4) are best heard with the bell of the stethoscope over the cardiac apex while the patient is in the left lateral decubitus position at the end of expiration. Valsalva strain phase increases intrathoracic pressure to compress the vena cava and reduce VR.

A 30-year-old man has a 4-day history of progressively worsening abdominal pain and bloody diarrhea. He was started on mesalamine therapy 6 months ago after being diagnosed with ulcerative colitis, but has been nonadherent to therapy. He is febrile, lethargic, and has dry mucous membranes. There is marked abdominal distention and tenderness without rebound or guarding. Rectal exam shows guaiac-positive, maroon-colored, and liquid stool. What is the next best step? A. Colonoscopy B. Plain abdominal x-ray

B. Plain abdominal x-ray Toxic megacolon is a well-recognized complication of ulcerative colitis. Patients typically present with abdominal pain/distention, bloody diarrhea, fever, and signs of shock. Plain abdomain lx-ray is the preferred diagnostic imaging study. Barium contrast studies and colonoscopy are contraindicated due to the risk of perforation.

A biochemist cultures hepatocytes with nitrogen-labeled glutamate. After some time, these isotypes are transferred to oxaloacetate, forming aspartate in the process. Which of the following was probably involved? A. Biotin B. Pyridoxine

B. Pyridoxine Transamination reactions typically occur between an amino acid and an alpha-keto acid. The amino group from the amino acid is transferred to the alpha-keto acid, and the alpha-keto acid in turn becomes an amino acid. Pyridoxal phosphate (active vitamin B6) serves as a cofactor in amino acid transamination and decarboxylation reactions.

45-year-old woman has colicky abdominal pain, diarrhea, and nausea 15-20 minutes after meals. These episodes are accompanied by light-headedness and diaphoresis. She feels normal between the episodes and at night. The patient underwent gastric bypass surgery 4 weeks ago for obesity. The patient has a well-healing midline abdominal scar. Mild epigastric tenderness to palpation is present. What is the most likely cause of their symptoms? A. Postsurgical scarring and stenosis of the gastrojejunostomy site B. Rapid transition of hyperosmolar food into the small bowel

B. Rapid transition of hyperosmolar food into the small bowel The pyloric sphincter prevents incompletely digested hyperosmolar food particles from entering the intestines too rapidly; when the sphincter is bypassed or surgically removed, dumping syndrome may occur. Symptoms are related to excessive intestinal fluid (abdominal cramps, diarrhea, nausea,) vasomotor hyperactivity (diaphoresis, palpitations, flushing) and intravascular volume depletion (hypotension, tachycardia).

A study is conducted to determine how dietary eating patterns influence total body iron stores in children. Researchers want to assess how 2 independent variables, red meat consumption and egg consumption, affect serum ferritin concentrations while adjusting for age and gender. Which statistical technique will determine this association? A. Relative risk B. Regression analysis

B. Regression analysis A regression analysis describes the effect that 1 or more independent variables (exposures, risk factors) which may be quantitative or qualitative, can have on 1 quantitative dependent variable (outcome). Odds ratio and relative risk measure the strength of association between 2 categorical variables. OR is uesd in case-control studies whereas the RR is used in cohort studies

A 26-year-old woman has new facial hair over the past few months and a deeper voice. Her last menstrural period was 5 months ago. The patient's mother died of infiltrating lobar breast carcinoma at age 60. Physical exam shows corase facial hair and clitoromegaly with a large adnexal mass. Urine pregnancy test is negative. US confirms a large ovariant mass. What is the most likely diagnosis? A. Granulosa cell tumor B. Sertoli-Leydig cell tumor

B. Sertoli-Leydig cell tumor Sertoli-Leydig cell tumors of the ovary arise from the sex cord stroma and secrete testosterone. Therefore, patients typically have signs and symptoms of hyperandrogenism, including rapid-onset virilization (hirsutism, clitoromegaly, voice deepening) and amenorrhea. Granulosa cell tumors are also sex cord stromal tumors, but they secrete estrogen and inhibin. Patients typically have breast tenderness, abnormal uterine bleeding due to endometrial hyperplasia from excess estrogen rather than signs of excess testosterone.

A 45-year-old woman comes in for a followup for type-II diabetes. When asked about her meds, she says "I use it most of the time, but its easy to forget when I am feeling good." What is the most appropriate response to discuss nonadherence? A. Its very important that you take your medication regularly, even when you feel fine B. Taking medications daily can be difficult, especially on days you feel well

B. Taking medications daily can be difficult, especially on days you feel well Physicians should use a nonjudgmental, patient-centered approach in discussing treatment nonadherence. Acknowledging the difficulty of taking medication regularly can stregthen the therapeutic alliance, potentially improving patient receptiveness to educational efforts. The incorrect answer gives the patient a directive without first exploring her perspective and understanding of the illness. The best initial approach is to engage the patient with a statement that validates her experience and encourages open discussion of her challenges with medication adherence.

A 19-year-old woman has 2 days of dysuria, urinary frequency, and urgency. The symptoms began after having sex with her boyfriend. They have no fever, flank pain, nausea, or vomiting. She has no drug allergies. Exam shows mild suprapubic tenderness. Urinalysis reveals many bacteria and white blood cells. What is the most appropriate therapy? A. Metronidazole B. Trimethoprim-Sulfamethoxazole

B. Trimethoprim-Sulfamethoxazole Acute simple cystitis often occurs in young, otherwise healthy women, particularly following sexual intercourse. Common manifestations include dysuria, urinary frequency/urgency, and urinalysis evidence of pyuria/bacteriuria. Empiric treatment with nitrofurantoin or trimethoprim-sulfamethoxazole is typically curvative. Metronidazole can treat Trich, which often presents with dysuria and frequency, but most also have a purulent, malodourous vaginal discharge. The presence of pyuria/bacteriuria on urinalysis makes acute bacterial cystitis more likely.

A 66-year-old man has numbness and tingling in his hands and feet. He was diagnosed with TB 3 months ago. He takes only isoniazid and rifampin. Neurologic exam shows ataxia and decreased pain sensation in the distal extremities. This patient's condition is most likely due to which of the following? A. Direct drug-mediated toxic neuron damage B. Vitamin-dependent neurotransmitter deficiency

B. Vitamin-dependent neurotransmitter deficiency Isoniazid is structurally similar to pyridoxine (vitamin B6) and competes for binding sites on pyridoxine-dependent enzymes. This leads to decreased synthesis of certain neurotransmitters, which may result in peripheral neuropathy. Management involves pyridoxine supplementation. Certain chemotherapeutic drugs (vincristine) may cause peripheral neuroapathy by inhibiting microtubule formation. In contrast, isoniazid is not directly toxic to neurons.

A 32-year-old with a 2-day history of worsening headache and double vision has nasal discharge for the past several days as well. Physical exam reveals weakening of the muslce shown here: https://i.imgur.com/2jpIYhs.png. Purulent discharge from the sphenoethmoidal recess of the nasal cavity is also noted. Involvement of which of the following nerves best explains this patient's ocular findings? A. optic nerve B. abducens nerve

B. abducens nerve The abducens nerve (CN VI) innervates the lateral rectus muscle, which originates at the annular tendon and inserst on the temporal (lateral) surface of the globe. It functions to abduct the eye.

A 58-year-old man has a headache in the ER. He has long-standing hypertension and medication nonadherence. BP is 231/135. 20 mins after hydralazine is administered IV, his BP drops to 145/95. Over the following 24 hours, serum creatinine increases from 1.1 to 1.6. Urine sediment demonstrates abundant granular casts. What best explains the patient's acute kidney injury? A. drug-induced renal interstitial inflammation B. alteration of the blood pressure flow relationship in renal arterioles

B. alteration of the blood pressure flow relationship in renal arterioles In patients with long-standing hypertension, a chronic autoregulatory shift in the blood-pressure flow relationship occurs, affording less perfusion (flow) at any given pressure. Therefore, excessively rapid correction of BP toward normal may induce relative ischemia. Drug -induced allergic interstitial nephritis is an eosinophilic AKI usually triggered by beta-lactam antibiotics and NSAIDs. It occurs after continual exposure (Several days) and eosinophiluria is usually present

A 12-year-old boy has chronic headaches and visual changes. The headaches have progressively worsened over the past year and are associated with nausea. An intracranial calcified mass is detected on brain imaging. The mass is surgically removed and on gross inspection shows cystic spaces filled with thick, brownish-yellow fluid rich in cholestered. The mass is most likely derived from cells responsible for forming which structure? A. choroid plexus B. anterior pituitary gland

B. anterior pituitary gland Craniopharyngiomas are suprasellar tumors found in children and composed of calcified cysts containing cholesterol crystals. They arise from remnants of Rathke's pouch, an embryonic precursor of the anterior pituitary. "motor oil" appearance

A study is performed to compare the effect of tramadol when compared to placebo for painful polyneuropathy. Fifty patients are selected and randomly allocated to 1 of 2 treatment sequences: Tramadol followed by placebo, or placebo followed by tramadol. Initial treatment period is delivered for 4 weeks followed by an interim 1-week washout phase, after which the second treatment period is delivered for an additional 4 weeks. After each treatment period, patients use a 10-point numerical scale to rate symptoms. What is the best description of this study design? A. case-control study B. crossover study

B. crossover study In crossover studies, subjects are randomly allocated to a sequence of 2 or more treatments given consecutively. A washout (no treatment) period is often added between treatment intervals to limit the confounding effects of prior treatment. A case-control study is designed by selecting patients with a particular disease (cases) and without that disease (controls), then determining their past exposure status to >1 risk factors believed to be associated with the disease of interest.

A 27-year-old woman is brought to the ER by her employer for increasingly paranoid and disorganized behavior. She has had difficult performing her work and has become increasingly secretive over the past 6 months, now talking in a whisper because she believes that listening devices are planted in the walls. She's been hiding food and personal possessions in closets and under beds. During the interview, she changes topics frequently and rambles about voices harrassing her. What additional mental status finding will you see in this patient? A. olfactory hallucinations B. decreased facial expressiveness

B. decreased facial expressiveness In addition to positive psychotic symptoms (delusions, hallucinations, disorganization), patients with schizophrenia frequently exhibit negative symptoms such as flat affect, (lack of facial expression). Negative symptoms typically persist between acute psychotic episodes and are more resistant to treatment. Olfactory/tactile hallucinations can occur in schizophrenia, but are uncommon and more in line with substance use disorders. Auditory hallucination is the most common type of hallucination in schizophrenia.

An 8-year-old boy was diagnosed with absence seizures a year ago and has been undergoing treatment with an appropriate antiepileptic agent. His mother says its been helping a lot, with no seizures since starting the medication, improved school performance, and no longer any trouble "staring off" in class. What is the most likely MoA of the medication used for this patient? A. binds to GABA receptors and increases chloride current B. decreases calcium current in thalamic neurons

B. decreases calcium current in thalamic neurons Ethosuximide, the first-line treatment for absence seizures, prevents neuronal burst firing by blocking thalamic T-type Ca channels. Phenytoin, carbamazepine, and valproic acid inhibit neurona high-frequency firing by reducing the ability of Na channels to recover from inactivation. Benzos and barbiturates (phenobarbital) bind GABA receptors and allow an increase in Cl current through the ion channel. This hyperpolarizes the cell membrane, which increases the threshold of excitability of the postsynaptic neuron.

A 70-year-old woman reports intermittent headaches and poor sleep. She has described a sense of incomplete emptying of the bladder and constipation. She has a history of migraines and chronic insomnia. Her other medical problems include hypercholesterolemia and osteoarthritis. Her medications include amitriptyline prescribed for migraines, simvastatin, diclofenac, and OTC diphenhydramine for insomnia. What is the next best step in her management? A. head CT scan B. discontinue diphenhydramine

B. discontinue diphenhydramine Cumulative anticholinergic burden of multiple medications is especially problematic in the elderly. A head CT scan would be extremely low yield in this patient with chronic headaches.

A 1-hour-old boy in a newborn nursey has jitteriness and irritability. He was born by C-section due to arrest of the second stage of labor. He has not yet breastfed. The pregnancy was complicated by gestational diabetes mellitus. The mother was prescribed insulin but did not take it regularly. Birth weight is 10 lbs. Cardiopulmonary exam is within normal limits. Serum glucose is 21 mg/dL. What is the cause of this neonate's hypoglycemia? A. decreased fetal hepatic glycogen concentration B. fetal pancreatic beta cell hyperplasia

B. fetal pancreatic beta cell hyperplasia Maternal hyperglycemia causes increased transplacental transfer of glucose to the infant, resulting in fetal hyperglycemia and subsequent pancreatic beta cell hyperplasia. The pancreatic beta cell hyperplasia causes hyperinsulinemia, leading to fetal macrosomia and neonatal hypoglycemia. My answer is wrong because insulin promotes blood glucose storage in the liver as glycogen, therefore infants of mothers with poorly controlled diabetes have increased glycogen stores. However, after birth these infants have poor mobilization of glycogen stores (glycogenesis) despite hypoglycemia due to high levels of insulin from islet hyperplasia.

A 64-year-old man comes to the ER with worsening dizziness and shortness of breath for several days. He was diagnosed with advanced stage lung cancer 6 months ago. BP is 85/45, pulse is 122, and RR is 22. Pulse ox is 94% on room air. There are undulations in BP at different phases of respiration. Maximal impulse is not palpable, heart sounds are distant. Xray reveals diffuse pulmonary metastatic disease and new enlargement of the cardiac silhouette. Which of the following hemodynamic changes are most likeky present due to this patient's cardiac condition? A. decreased CVP, normal right atrial cavity size, decreased LAP, decreased SV B. increased CVP, decreased right atrial cavity size, increased LAP, decreased SV

B. increased CVP, decreased right atrial cavity size, increased LAP, decreased SV Cardiac tamponade signs: hypotension, distant heart sounds, new enlargement of cardiac silhouette in the setting of advanced malignancy. It causes obstructive shock, Beck's triad (hypotension, distant heart sounds, JVD), absent point of maximum impulse, and pulsus paradoxus (10 mmHg drop in systolic BP with inspiration). Cardiac tamponade involves increased pericardial pressure that restricts diaostlic filling of the right-sided heart chambers, leading to obstructive shock with increased central venous pressure (CVP) and decreased stroke volume (SV) and cardiac ouput. As tamponade progresses, diastolic pressure in all 4 cardiac chambers increase and equalize with pericardial pressure (and with one another), stagnating blood flow within the heart. Left atrial pressure increases due to external compression by excessive pericardial fluid.

A 14-year-old girl has a murmur best heard at the left upper sternal border. Phonograph reveals a widely-split S2 that is fixed (no variation between inspiration and expiration). It is most likely due to which of the following? (pic of sonogram: https://i.imgur.com/XZOUk1I.png) A. regurgitation of blood from the aorta B. increased blood flow to the lungs

B. increased blood flow to the lungs An atrial septal defect leads to increased right-sided blood flow due to left-to-right shunting. Characteristics findings are an early-peaking midsystolic ejection murmur over the left upper sternal border (pulmonic flow murmur) and a wide and fixed splitting of S2. Aortic insufficiency is characterized by regurgitant blood flow through the aortic valve, resulting in a decrescendo diastolic murmur along the sternal border. Splitting of S2 is not usually affected.

A 64-year-old man has a lump in his neck. He has a long history of chewing tobacco. On exam there is an ulcerative lesion in the left floor of the mouth. There is also a firm, palpable, nontender left submandibular mass. Biopsy of the submandibular mass shows atypical mitotic figures with keratin pearls (https://i.imgur.com/qOU5d9v.png). What is the most likely route of spread from the floor of the mouth to the submandibular mass? A. direct invasion through the floor of the mouth B. lymphatic drainage to the submandibular lymph nodes

B. lymphatic drainage to the submandibular lymph nodes Head and neck squamous cell carcinomas typically spread first to the anterior cervical lymph nodes via the lymphatics. The involvement of anterior cervical lymph nodes affects the staging of the disease.

A 42-year-old man complains of abdominal pain, vomiting, and severe watery diarrhea. He recently returned from a camping trip and admits to eating wild mushrooms. He is ill-appearing and jaundiced. His liver edge is soft, tender, and palpable 4 cm below the right costal margin. Lab tests show elevated AAT, AST, and bilirubin. Synthesis of which of the following is most likely to be directly inhibited by the responsible toxin? A. Protein B. mRNA

B. mRNA Amatoins are found in a variety of poisonous mushrooms and are potent inhibitors of RNA polymerase II, which halts mRNA synthesis. Ricin is a potent toxin that inhibits protein synthesis by cleaving rRNA component of 60S subunit

A neonate has increased TSH and low thyroxine (T4). He was born by vaginal delivery. Exam shows enlarged thyroid gland in normal posiiton, confirmed by neck US. What is the most likely cause? A. transplacental passage of thyroglobulin B. maternal intake of propylthiouracil

B. maternal intake of propylthiouracil Congenital goiter in the setting of hypothyroidism (high TSH, low thyroxine) can be caused by transplacental passage of maternal antithyroid medications (eg propylthiouracil), which inhibit thyroid peroxidase and are used to treat maternal hyperthyroidism. Levothyroxine like other thyroid hormones, does not readily cross the placenta. In addition, this medication treats, not causes, hypothyroidism

A 22-year-old woman comes to the office after several years of persistent facial blemishing that has failed to respond to OTC treatment. Skin exam: https://i.imgur.com/IIdGjvz.png. Which of the following most likely contributed to this? A. androgen-induced involution of sebaceous glands B. proliferation of lipid-utilizing bacteria within pilosebaceous follicles

B. proliferation of lipid-utilizing bacteria within pilosebaceous follicles Acne is characterized by obstruction of pilosebaceous follicles (ie comedones) due to hyperkeritanization and excessive sodium accumulation. Androgens stimulate production of sebum, which serves as a nutrient source for Cutibacterium acnes. Within the follicles, C acnes proliferation triggers an inflammatory response, resulting in formation of red papules and pustules. In response to androgen stimulation (pubertal adrenarche), sebaceous glands enlarge (NOT involute) and increase production of sebum, a lipid-rich substance that facilitates obstruction of pilosebaceous follicles.

A 23-year-old man has sudden onset heart palpitations. His BP is 110/70 and pulse is 160 and regular. Gentle neck massage just below the angle of the right mandible provides immediate improvement of his condition to BP 120/80 and pulse 75. What mechanism is responsible for improvement of this patient's condition? A. increased systemic vascular resistance B. prolonged AV node refractory period

B. prolonged AV node refractory period Carotid sinus massage leads to an increase in PSNS tone causing temporary inhibition of SA node activity, slowing conduction through the AV node, and prolongation of the AV node refractory period. It is a useful vagal maneuver for termination of paroxysmal supraventricular tachycardia. Carotid sinus massage stimulates the baroreceptors and increases the firing rate from the carotid sinus, leading to an increased PSNS output and withdrawal of SNS output to the heart and peripheral vasculature. This leads to slowing of HR and AV conduction along with DECREASED SVR.

A 23-year-old man has abrupt-onset high fever, chills, headache, and weakness that developed after working a month on his large ranch. His initial symptoms were quicky followed by intensely painful swellings in the groin. Physical exam shows tender and enlarged inguinal lymph nodes with overlying erythematous skin. Blood cultures and fluid aspirated from the lymph nodes grow gram-negative coccobacilli that resembe closed safety pins on special staining. Which of the following activities is the most likely source of this patient's infection? A. hunting rabbits B. replacing rodent traps

B. replacing rodent traps Bubonic plague causes a febrile illness with regional inflammatory lymphadenopathy (buboes). The underlying pathogen is Yersinia pestis, a small gram-negative bacillus/coccobacillus that exhibits bipolar staining (resembling a closed safety pin). The major environmental reservoir is rodents; transmission usually occurs via rodent fleabite. Hunting rabbits is a common cause of tularemia, which often presents with painful regional lymphadenitis and fever. However, patients usually develop a papular lesion at the inoculation site that ulcerates. Although Francisella tularensis, the causative pathogen, is also a gram-negative coccobacillus, it is not associated with bipolar staining and a safety pin appearance.

A 19-year-old man has uncharacteristic behavior. A year ago, his father died unexpectedly in an MVC. Afterwards, the patient became increasingly withdrawn and stopped responding to text messages. He became disinterested in his schoolwork and did not graduate from high school due to failing grades. For the past few months he has spent most of his time in his room along listening to music and drawing. His mother says his drawings are all of numbers with different colors and sizes. He's been fixated on the date and time of his father's death and thinks these numbers have a special meaning. He is difficult to follow in conversation, changing topics frequently with random remarks. On mental status exam, he pauses before answering questions and provides brief, unrelated responses. He does not make eye contact and his affect is blunted. He has no suicidal ideation or hallucinations. What is the most likely diagnosis? A. PTSD B. schizophrenia

B. schizophrenia Schizophrenia is diagnosed in patients with signs of disturbance for >6months, including at least 1 month of >2 of the following active symptoms (which at least 1 from the first 3): delusions, hallucinations, disorganized speech, disorganized/catatonic behavior, and negative symptoms. PTSD involves psychological distress following a traumatic event. It can present with distress-related disturbances resembling hallucinations and/or mood alterations with anhedonia and negative emotionality. However, the level of disorganized, incoherent speech, and thought in this patient are not seen in PTSD.

An 18-year-old woman has low energy, decreased appetite, insomnia, poor concentration, and decreased interest in her daily activities. She has los 8 lb over the last month. Medication is administered, and 3 weeks later she is brought back to the ER for not sleeping at all for 3 nights. The patient reports that she does not feel tired despite lack of sleep. She spent a large amount of money on plane tickets and plans to travel around the world to promote world peace. Her mothers adds that she has suffered from severe mood swings in the past. What was she most likely started on 3 weeks ago? A. lithium B. sertraline

B. sertraline Antidepressant monotherapy can induce mania in susceptible patients, especially those with unrecognized bipolar disorder. Carbamazepine, lithium, and valproate are mood stabilizers used in acute episodes and maintenance treatment of bipolar disorder. The patient presented with depression with no known history of bipolar at the time, so a mood stabilizer would not be the initial first-line treatment.

A 54-year-old man is transferred to the hospital due to an Afib with rapid ventricular response. His symptoms began 24 hours earlier with palpitations, lightheadedness, and chest tightness. The patient has had similar episodes several times during the last year. Past history is notable for mitral valve repair 3 years ago for severe symptomatic mitral regurgitation. The patient undergoes direct cardioversion due to worsening hemodynamic instability with resolution of arrythmia and all associated symptoms. He is considered for long-term amiodarone therapy to prevent future arrythmic episodes. What should be tested before initiation amiodarone therapy? A. oral glucose tolerance test B. serum TSH

B. serum TSH Amiodarone is 40% iodine by weight. It can cause hypothyroidism due to decreased production of the thyroid hormone. Amiodarone can also cause hyperthyroidism due to increased thyroid hormone synthesis or destructive thyroiditis with release of preformed thyroid hormone. The oral glucose tolerance test is used to screen for diabetes mellitus, primarily in pregnant women. Amiodarone does not cause significant alterations in insulin sensitivity or blood glucose levels.


Related study sets

Networking + Post-Assessment Quiz

View Set

Financial Leverage and ROE Analysis

View Set

MGMT 320 Human Resource Management Study Vocab Exam 1

View Set